SlideShare uma empresa Scribd logo
1 de 36
Baixar para ler offline
OOOOBBBBJJJJEEEETTTTIIIIVVVVOOOO
QQQQUUUUÍÍÍÍMMMMIIIICCCCAAAA
CONSTANTES
Constante de Avogadro = 6,02 x 1023 mol–1
Constante de Faraday (F) = 9,65 x 104C mol–1
Volume molar de gás ideal = 22,4 L (CNTP)
Carga elementar = 1,602 x 10–19 C
Constante dos gases (R) =
= 8,21 x 10–2 atm L K–1 moI–1 = 8,31 J K–1 moI–1 =
= 62,4 mmHg L K–1 mol–1 = 1,98 cal K–1 mol–1
DEFINIÇÕES
Condições normais de temperatura e pressão
(CNTP): 0°C e 760 mmHg.
Condições ambientes: 25°C e 1 atm.
Condições-padrão: 25°C, 1 atm, concentração das so-
luções: 1 mol L–1 (rigorosamente: atividade unitária das
espécies), sólido com estrutura cristalina mais estável
nas condições de pressão e temperatura em questão.
(s) ou (c) = sólido cristalino; (l) ou (l) = líquido; (g) = gás;
(aq) = aquoso; (graf) = grafite; (CM) = circuito metálico;
(conc) = concentrado; (ua) = unidades arbitrárias;
[A] = concentração da espécie química A em mol L–1.
MASSAS MOLARES
Massa Molar
(g mol–1
)
1,01
4,00
6,94
9,01
10,81
12,01
14,01
16,00
19,00
22,99
28,09
30,97
32,06
35,45
Número
Atômico
1
2
3
4
5
6
7
8
9
11
14
15
16
17
Elemento
Químico
H
He
Li
Be
B
C
N
O
F
Na
Si
P
S
Cl
IIIITTTTAAAA ---- ((((4444ºººº DDDDiiiiaaaa)))) DDDDeeeezzzzeeeemmmmbbbbrrrroooo////2222000000005555
OOOOBBBBJJJJEEEETTTTIIIIVVVVOOOO
As questões de 01 a 20 NÃO devem ser resolvidas
no caderno de soluções. Para respondê-Ias, marque a
opção escolhida para cada questão na folha de leitura
óptica e na reprodução da folha de leitura óptica
(que se encontra na última página do caderno de
soluções).
1 AAAA
Considere uma amostra nas condições ambientes que
contém uma mistura racêmica constituída das subs-
tâncias dextrógira e levógira do tartarato duplo de sódio
e amônio.
Assinale a opção que contém o método mais adequado
para a separação destas substâncias.
a) Catação. b) Filtração. c) Destilação.
d) Centrifugação. e) Levigação.
Resolução
Isômeros ópticos (dextrogiros e levogiros) desviam o
plano de vibração da luz polarizada.
Uma mistura racêmica é constituída por partes iguais
dos isômeros dextrogiro e levogiro. As propriedades
físicas desses isômeros são iguais e, portanto, a única
maneira de separá-los, dentre as alternativas citadas,
seria por catação por formarem cristais diferentes em
suas formas enantiomorfas.
Esses cristais poderiam ser separados por uma pinça.
Os isômeros poderiam também ser separados por
processos químicos.
Massa Molar
(g mol–1
)
39,10
40,08
52,00
54,94
55,85
58,69
63,55
79,91
83,80
107,87
118,71
126,90
200,59
207,21
Número
Atômico
19
20
24
25
26
28
29
35
36
47
50
53
80
82
Elemento
Químico
K
Ca
Cr
Mn
Fe
Ni
Cu
Br
Kr
Ag
Sn
I
Hg
Pb
IIIITTTTAAAA ---- ((((4444ºººº DDDDiiiiaaaa)))) DDDDeeeezzzzeeeemmmmbbbbrrrroooo////2222000000005555
OOOOBBBBJJJJEEEETTTTIIIIVVVVOOOO
2 DDDD
Considere os seguintes óxidos (I, ll, III, IV e V):
I. CaO lI. N2O5 III.Na2O IV. P2O5 V. SO3
Assinale a opção que apresenta os óxidos que, quando
dissolvidos em água pura, tornam o meio ácido.
a) Apenas I e IV b) Apenas I, III e V
c) Apenas II e III d) Apenas lI, IV e V
e) Apenas III e V
Resolução
I) CaO + H2O → Ca(OH)2, meio básico
II) N2O5 + H2O → 2HNO3, meio ácido
III) Na2O + H2O → 2NaOH, meio básico
IV) P2O5 + 3H2O → 2H3PO4, meio ácido
V) SO3 + H2O → H2SO4, meio ácido
3 BBBB
Assinale a opção que apresenta a equação química que
representa uma reação envolvendo a uréia [CO(NH2)2]
que NÃO ocorre sob aquecimento a 90 °C e pressão de
1 atm.
a) CO(NH2)2(s) + 2HNO2(aq) →
→ 2N2(g) + CO2(g) + 3H2O(l)
b) CO(NH2)2(s) → N2(g) + 1/2 O2(g) + CH4(g)
c) CO(NH2)2(s) + H2O(l) → 2NH3(g) + CO2(g)
d) CO(NH2)2(s) + H2O(l) + 2HCl(aq) →
→ 2NH4Cl(aq) + CO2(g)
e) CO(NH2)2(s) + 2 NaOH (aq) → Na2CO3(aq) + 2NH3(g)
Resolução
Uréia é fracamente básica e forma sais com os ácidos
fortes. É hidrolisada em presença de ácidos, álcalis ou
da enzima urease.
a) Ocorre. A uréia reage com ácido nitroso liberando
gás nitrogênio.
b) Não ocorre, pois a uréia é um composto estável.
Ela se decompõe produzindo NH3 e CO2 na forma
de equilíbrio: 2NH3 + CO2
→
← CO(NH2)2 + H2O
c) Ocorre. Hidrolisa formando amônia e gás carbô-
nico.
H2O
CO(NH2)2(s) → 2NH3(g) + CO2(g)
d) Ocorre. Hidrolisa em meio ácido formando íon
amônio e gás carbônico.
H2O
CO(NH2)2(s) → NH4
+
(aq) + CO2(g)
H+
e) Ocorre. Hidrolisa em meio alcalino formando
carbonato e amônia.
H2O
CO(NH2)2(s) → CO3
2–
+ NH3
OH–
IIIITTTTAAAA ---- ((((4444ºººº DDDDiiiiaaaa)))) DDDDeeeezzzzeeeemmmmbbbbrrrroooo////2222000000005555
OOOOBBBBJJJJEEEETTTTIIIIVVVVOOOO
4 DDDD
São fornecidas as seguintes informações a respeito de
titulação ácido-base:
a) A figura mostra as curvas de titulação de 30,0 mL de
diferentes ácidos (I, ll, llI, IV e V), todos a 0,10 mol L–1,
com uma solução aquosa 0,10 moI L–1 em NaOH.
b) O indicador fenolftaleína apresenta o intervalo de
mudança de cor entre pH 8,0 a 10,0, e o indicador
vermelho de metiIa, entre pH 4,0 a 6,0.
Considerando estas informações, é CORRETO afirmar
que
a) o indicador vermelho de metila é mais adequado que
a fenolftaleína para ser utilizado na titulação do ácido
IV.
b) o indicador vermelho de metila é mais adequado que
a fenolftaleína para ser utilizado na titulação do ácido
V.
c) o ácido III é mais forte que o ácido ll.
d) os dois indicadores (fenolftaleína e vermelho de
metila) são adequados para a titulação do ácido I.
e) os dois indicadores (fenolftaleína e vermelho de
metila) são adequados para a titulação do ácido llI.
Resolução
Dentre os cinco ácidos citados, o mais forte é o I, pois
para uma mesma concentração (0,10 mol L–1) é o que
apresenta menor pH (maior concentração de íons H+):
I > II > III > IV > V
A neutralização de um ácido forte com uma base forte
(NaOH) produz um sal que em solução resulta num
meio neutro, com pH próximo de 7 (25°C).
Se neutralizarmos um ácido fraco com uma base forte
(NaOH), o meio resultante será básico e, nesse caso,
devemos utilizar um indicador com pH de viragem aci-
ma de 7 (somente a fenolftaleína).
Exceção feita ao ácido I, podemos observar que os
demais ácidos apresentam ponto de equivalência em
meio de pH acima de 7.
Para o ácido I, que apresenta pH de equivalência igual a
7, além da fenolftaleína, podemos usar como indicador
o vermelho-de-metila.
IIIITTTTAAAA ---- ((((4444ºººº DDDDiiiiaaaa)))) DDDDeeeezzzzeeeemmmmbbbbrrrroooo////2222000000005555
OOOOBBBBJJJJEEEETTTTIIIIVVVVOOOO
5 CCCC
Considere as seguintes afirmações a respeito da varia-
ção, em módulo, da entalpia (∆H) e da energia interna
(∆U) das reações químicas, respectivamente represen-
tadas pelas equações químicas abaixo, cada uma
mantida a temperatura e pressão constantes:
I. H2O(g) + 1/2O2(g) → H2O2(g) ; |∆HI| > |∆UI|
ll. 4NH3(g) + N2(g) → 3N2H4(g) ; |∆HII| < |∆UII|
llI. H2(g) + F2(g) → 2HF(g) ; |∆HIII| > |∆UIII|
IV.HCl(g) + 2O2(g) → HClO4(l) ; |∆HIV| < |∆UIV|
V. CaO(s) + 3C(s) → CO(g) + CaC2(s); |∆HV| > |∆UV|
Das afirmações acima, estão CORRETAS
a) apenas I, II e V. b) apenas I, III e IV.
c) apenas ll, IV e V. d) apenas III e V.
e) todas.
Resolução
A relação entre variação de entalpia (∆H) e da energia
interna (∆U) é:
∆H = ∆U + τ
onde τ = ∆n R T
∆n = variação da quantidade de mols dos gases
I) H2O(g) + O2(g) → H2O2(g)
1,5 mol 1 mol
∆HI = ∆UI – 0,5RT
͉∆UI ͉ > ͉∆HI ͉
II) 4NH3(g) + N2(g) → 3N2H4(g)
5 mol 3 mol
∆HII = ∆UII – 2RT
͉∆UII͉ > ͉∆HII͉
III) H2(g) + F2(g) → 2HF(g)
2 mol 2 mol
͉∆HIII͉ = ͉∆UIII͉
IV) HCl(g) + 2O2(g) → HClO4(l)
3 mol 0 mol de gás
∆HIV = ∆UIV – 3RT
͉∆UIV ͉ > ͉∆HIV ͉
V) CaO(s) + 3C(s) → CO(g) + CaC2(s)
0 mol de gás 1 mol
∆HV = ∆UV + 1RT
͉∆HV ͉ > ͉∆UV ͉
Corretas: II, IV e V
1
–––
2
IIIITTTTAAAA ---- ((((4444ºººº DDDDiiiiaaaa)))) DDDDeeeezzzzeeeemmmmbbbbrrrroooo////2222000000005555
OOOOBBBBJJJJEEEETTTTIIIIVVVVOOOO
6 AAAA
Considere as afirmações abaixo, todas relativas à tem-
peratura de 25°C, sabendo que os produtos de
solubilidade das substâncias hipotéticas XY, XZ e XW
são, respectivamente, iguais a 10–8, 10–12 e 10–16,
naquela temperatura.
I. Adicionando-se 1 x 10–3 mol do ânion W proveniente
de um sal solúvel a 100 mL de uma solução aquosa
saturada em XY sem corpo de fundo, observa-se a
formação de um sólido.
ll. Adicionando-se 1 x 10–3 mol do ânion Y proveniente
de um sal solúvel a 100 mL de uma solução aquosa
saturada em XW sem corpo de fundo, não se
observa a formação de sólido.
llI. Adicionando-se 1 x 10–3 mol de XZ sólido a 100 mL
de uma solução aquosa contendo 1 x 10–3 moI L–1 de
um ânion Z proveniente de um sal solúvel, observa-
se um aumento da quantidade de sólido.
IV.Adicionando-se uma solução aquosa saturada em XZ
sem corpo de fundo a uma solução aquosa saturada
em XZ sem corpo de fundo, observa-se a formação
de um sólido.
Das afirmações acima, está(ão) CORRETA(S)
a) apenas I e ll. b) apenas I e llI.
c) apenas ll. d) apenas III e IV.
e) apenas IV.
Resolução
Equações de dissociação das substâncias hipotéticas:
XY(s) →← X +
(aq) + Y –
(aq) KPS = [X+] [Y –] = 10–8
XZ(s) →← X +
(aq) + Z –
(aq) KPS = [X+] [Z –] = 10–12
XW(s) →← X +
(aq) + W –
(aq) KPS = [X+] [W –] = 10–16
I) Correta.
Cálculo da concentração de X +
na solução XY:
KPS = [X+] [Y –]
10 –8 = x . x
x = ͙ෆෆෆ10 –8
x = 10–4 ⇒ [X+] = 10 –4 mol/L
Cálculo da concentração de W –
:
[W –
] = = 10 –2 mol/L
X+
+ W – →← X W
KPS
XW
= [X +
] [W –
] = 10–16
Quando [X+
] . [W –
] > 10–16
, temos precipitação:
[X+
] . [W –
] = 10–4
. 10–2
= 10–6
> KPS (formação de
sólido)
II) Correta.
Cálculo da concentração de X +
na solução XW:
KPS = [X+] [W–]
10 –16 = x . x
x = ͙ෆෆෆ10 –16
1 . 10–3 mol
–––––––––––
10–1L
IIIITTTTAAAA ---- ((((4444ºººº DDDDiiiiaaaa)))) DDDDeeeezzzzeeeemmmmbbbbrrrroooo////2222000000005555
OOOOBBBBJJJJEEEETTTTIIIIVVVVOOOO
x = 10–8 ⇒ [X+] = 10 –8 mol/L
Cálculo da concentração de Y –
:
[Y –
] = = 10 –2 mol/L
X+
+ Y – →← X Y
KPS
XY
= [X +
] [Y –
] = 10–8
Quando [X+
] . [Y –
] > 10–8
, temos precipitação:
[X+
] . [Y –
] = 10–8
. 10–2
= 10–10
< KPS (não temos
formação de
sólido)
III)Falsa.
Cálculo da quantidade, em mol, de X+
e Z–
no sólido:
XZ → X +
+ Z –
1 . 10 –3 mol 1 . 10 –3 mol 1 . 10 –3 mol
Cálculo da quantidade, em mol, de Z –
na solução:
1L ––––– 1 . 10–3 mol
}a = 0,1 . 10–3
mol
0,1L ––––– a
Quantidade total de Z –
: 1,1 . 10–3
mol
Equação da reação de precipitação:
X+
(aq) + Z–
(aq) → XZ(s)
1 . 10 –3 mol 1,1 . 10 –3 mol 1 . 10 –3 mol
A quantidade de sólido inicial é praticamente igual à
quantidade de sólido final, porque a solubilidade é
muito pequena (10–6 mol/L).
IV)Falsa.
Quando se misturam soluções de mesma con-
centração, saturadas sem corpo de fundo, de
mesmo soluto, a solução resultante apresentará a
mesma concentração, portanto a solução não apre-
sentará corpo de fundo.
10–3 mol
–––––––––
10–1L
IIIITTTTAAAA ---- ((((4444ºººº DDDDiiiiaaaa)))) DDDDeeeezzzzeeeemmmmbbbbrrrroooo////2222000000005555
OOOOBBBBJJJJEEEETTTTIIIIVVVVOOOO
7 AAAA
O diagrama de fases da água está representado na
figura. Os pontos indicados (I, ll, llI, IV e V) referem-se a
sistemas contendo uma mesma massa de água líquida
pura em equilíbrio com a(s) eventual(ais) fase(s)
termodinamicamente estável(eis) em cada situação.
Considere, quando for o caso, que os volumes iniciais
da fase vapor são iguais. A seguir, mantendo-se as
temperaturas de cada sistema constantes, a pressão é
reduzida até Pf. Com base nestas informações, assinale
a opção que apresenta a relação ERRADA entre os
números de moI de vapor de água (n) presentes nos
sistemas, quando a pressão é igual a Pf.
a) nI < nIII b) nl < nlV c) nIII < nll
d) nIII < nV e) nlV < nV
Resolução
Ponto I ⇒ apenas líquido
Ponto II ⇒ equilíbrio sólido, líquido e vapor
Ponto III ⇒ apenas líquido
Ponto IV ⇒ equilíbrio líquido-vapor
Ponto V ⇒ equilíbrio líquido-vapor
A quantidade em mol de líquido em todos os sistemas
é a mesma e igual a n.
No início, a quantidade em mol para o vapor tem a se-
guinte relação:
n’V
> n’IV > n’II
, pois TV > TIV > TII
Todos os sistemas sofrem uma expansão isotérmica
até Pf , na qual todos apresentam apenas a fase de
vapor.
Assim, no final, tem-se:
nI = n
nII = n + x + n’II
nIII = n
nIV = n + n’IV
nV = n + n’V
a) Falsa, pois nI = nIII
b) Correta, pois nI < nIV
c) Correta, pois nIII < nII
d) Correta, pois nIII < nV
e) Correta, pois nIV < nV
IIIITTTTAAAA ---- ((((4444ºººº DDDDiiiiaaaa)))) DDDDeeeezzzzeeeemmmmbbbbrrrroooo////2222000000005555
OOOOBBBBJJJJEEEETTTTIIIIVVVVOOOO
8 BBBB
Considere um calorímetro adiabático e isotérmico, em
que a temperatura é mantida rigorosamente constante e
igual a 40°C. No interior deste calorímetro é posi-
cionado um frasco de reação cujas paredes permitem a
completa e imediata troca de calor. O frasco de reação
contém 100 g de água pura a 40°C. Realizam-se cinco
experimentos, adicionando uma massa m1 de um sal X
ao frasco de reação. Após o estabelecimento do
equilíbrio termodinâmico, adiciona-se ao mesmo frasco
uma massa m2 de um sal Y e mede-se a variação de
entalpia de dissolução (∆H). Utilizando estas
informações e as curvas de solubilidade apresentadas
na figura, excluindo quaisquer condições de metaes-
tabilidade, assinale a opção que apresenta a correlação
CORRETA entre as condições em que cada expe-
rimento foi realizado e o respectivo ∆H.
a) Experimento 1: X = KNO3 ; m1 = 60 g; Y = KNO3 ;
m2 = 60 g; ∆H > 0.
b) Experimento 2: X = NaClO3; m1 = 40 g; Y = NaClO3;
m2 = 40 g; ∆H > 0
c) Experimento 3: X = NaCl ; m1 = 10 g; Y = NaCl; m2
= 10 g; ∆H < 0
d) Experimento 4: X = KNO3 ; m1 = 60 g; Y = NaClO3;
m2 = 60 g; ∆H = 0
e) Experimento 5: X = KNO3 ; m1 = 60 g; Y = NH4Cl;
m2= 60 g; ∆H < 0
Resolução
Um frasco contém 100g de água a 40°C. Adiciona-se
uma massa m1 de um sal X ao frasco e, depois de
estabelecido o equilíbrio, uma massa m2 de sal Y é
adicionada. Mede-se o ∆H de dissolução.
Experimento 1. Incorreta:
A solubilidade de KNO3 é 60g/100g de H2O a 40°C
Sendo: X = KNO3 e m1 = 60g → solução saturada
∆H > 0
Y = KNO3 e m2 = 60g → não será dissolvido
IIIITTTTAAAA ---- ((((4444ºººº DDDDiiiiaaaa)))) DDDDeeeezzzzeeeemmmmbbbbrrrroooo////2222000000005555
OOOOBBBBJJJJEEEETTTTIIIIVVVVOOOO
Experimento 2. Correta:
A solubilidade de NaClO3 é 120g/100g de H2O a 40°C
Sendo: X = NaClO3 e m1 = 40g → solução insaturada
∆H > 0
Y = NaClO3 e m2 = 40g → solução insaturada
∆H > 0
Portanto, todo o sal acrescentado dissolve-se.
Experimento 3. Incorreta:
A solubilidade de NaCl é aproximadamente 30g/100g
de H2O a 40°C
Sendo: X = NaCl e m1 = 10g → solução insaturada
∆H ≅ 0
Y = NaCl e m2 = 10g → solução insaturada
∆H ≅ 0
Toda a massa de sal dissolve-se e o ∆H é aproxi-
madamente zero.
Experimento 4. Incorreta:
Sendo: X = KNO3 e m1 = 60g → solução saturada
∆H > 0
Y = NaClO3 e m2 = 60g → solução insaturada
∆H > 0
Toda a massa de sal adicionado dissolve-se e o ∆H de
dissolução é maior que zero.
Experimento 5. Incorreta:
A solubilidade de NH4Cl é aproximadamente 40g/100g
de H2O a 40°C
Sendo: X = KNO3 e m1 = 60g → solução saturada
∆H > 0
Y = NH4Cl e m2 = 60g → solução saturada,
com excesso ∆H > 0
IIIITTTTAAAA ---- ((((4444ºººº DDDDiiiiaaaa)))) DDDDeeeezzzzeeeemmmmbbbbrrrroooo////2222000000005555
OOOOBBBBJJJJEEEETTTTIIIIVVVVOOOO
9 BBBB
A figura mostra cinco curvas de distribuição de velo-
cidade molecular para diferentes gases (I, II, III, IV e V)
a uma dada temperatura.
Assinale a opção que relaciona CORRETAMENTE a
curva de distribuição de velocidade molecular a cada
um dos gases.
a) I = H2 , II = He , III = O2 , IV = N2 e V = H2O.
b) I = O2 , II = N2, III = H2O , IV = He e V = H2.
c) I = He , II = H2 , III = N2 , IV= O2 e V = H2O.
d) I = N2 , II = O2 , III = H2 , IV = H2O e V = He.
e) I = H2O, II = N2 , III = O2 , IV = H2 e V = He.
Resolução
Quanto maior a massa molecular do gás, menor a
velocidade média molecular.
MMO2
> MMN2
> MMH
2
O > MMHe > MMH2
Foram dadas as curvas de Gauss de distribuição de
velocidade. Podemos concluir que
vO2
< vN2
< vH
2
O < vHe < vH2
I → O2 II → N2 III → H2O
IV → He V → H2
IIIITTTTAAAA ---- ((((4444ºººº DDDDiiiiaaaa)))) DDDDeeeezzzzeeeemmmmbbbbrrrroooo////2222000000005555
OOOOBBBBJJJJEEEETTTTIIIIVVVVOOOO
10 EEEE
Considere as respectivas reações químicas represen-
tadas pelas seguintes equações químicas:
I. 2KMnO4 +3H2SO4 + 5H2O2 → 1X + 2Y + 8H2O + 5O2
II. 4 CrO3 + 6 H2SO4 → 2Z +6 H2O + 3O2
III.2K2Cr2O7+10H2SO4→4KHSO4+2W + 8H2O + 3O2
Em relação às equações químicas I, II e III é CORRETO
afirmar que
a) o produto X é KHSO4.
b) o produto Y é Mn(SO4)2.
c) o produto Z é CrSO4.
d) o peróxido de hidrogênio atua como agente oxi-
dante.
e) os produtos Z e W representam o mesmo composto
químico.
Resolução
I) 2KMnO4 + 3H2SO4 +
1– 0
+ 5H2O2 → K2SO4 + 2MnSO4 + 8H2O + 5O2
X Y
oxidação
H2O2: agente redutor
II) 4CrO3 + 6H2SO4 → 2Cr2(SO4)3 + 6H2O + 3O2
Z
III) 2K2Cr2O7 + 10H2SO4 →
→ 4KHSO4 + 2Cr2(SO4)3 + 8H2O + 3O2
W
Os produtos Z e W representam o mesmo composto.
IIIITTTTAAAA ---- ((((4444ºººº DDDDiiiiaaaa)))) DDDDeeeezzzzeeeemmmmbbbbrrrroooo////2222000000005555
OOOOBBBBJJJJEEEETTTTIIIIVVVVOOOO
11 DDDD
Assinale a opção que apresenta a substância que pode
exibir comportamento de cristal líquido, nas condições
ambientes.
Resolução
No estado cristalino líquido, existe uma ordem mole-
cular menor do que num sólido, no entanto, maior do
que num líquido comum.
Compostos que podem formar cristais líquidos têm
moléculas compridas e razoavelmente rígidas.
A substância que pode exibir comportamento de cristal
líquido, nas condições ambientes, é:
A substância da alternativa e, embora seja uma molé-
cula de cadeia longa, tem menos interações dipolo-
dipolo.
CH3O — — CH = N — — CH2 — CH2 — CH2 — CH3
COOH
COOCH3
CH3O — — CH = N — — CH2 — CH2 — CH2 — CH3d)
CH3 — CH2 — CH2 — CH2 — CH2 — CH2 — CH2 — CH2 — CH2 — CH2 — OHe)
c)
— CH2COONaa)
b) CH3 — CH2 — C — CH2 — C — CH2 — CH3
CH3
CH3
CH3
CH3
IIIITTTTAAAA ---- ((((4444ºººº DDDDiiiiaaaa)))) DDDDeeeezzzzeeeemmmmbbbbrrrroooo////2222000000005555
OOOOBBBBJJJJEEEETTTTIIIIVVVVOOOO
12 CCCC
Considere quatro séries de experimentos em que
quatro espécies químicas (X, Y, Z e W) reagem entre si,
à pressão e temperatura constantes. Em cada série,
fixam-se as concentrações de três espécies e varia-se
a concentração (C0) da quarta. Para cada série, de-
termina-se a velocidade inicial da reação (v0) em cada
experimento. Os resultados de cada série são apre-
sentados na figura, indicados pelas curvas X, Y, Z e W,
respectivamente. Com base nas informações for-
necidas, assinale a opção que apresenta o valor
CORRETO da ordem global da reação química.
a) 3 b) 4 c) 5 d) 6 e) 7
Resolução
Considerando a reação
X + Y + Z + W → produtos, a equação da velocidade
pode ser expressa por:
v = k [X]a . [Y]b . [Z]c
. [W]d
Aplicando logaritmo em ambos os membros, temos:
log v = log k + a log [X] + b log [Y] + c log [Z] + d log [W]
Fixando-se as concentrações de três espécies e
variando a concentração da quarta espécie poderíamos
escrever 4 equações:
I) log v = a log [X] + k1
II) log v = b log [Y] + k2
III) log v = c log [Z] + k3
IV) log v = d log [W] + k4
k1 = k2 = k3 = – 1,0; k4 = – 0,8
Nas equações, a, b, c, d, são os coeficientes angulares
das 4 curvas fornecidas no gráfico log v0 x log C0.
Para X, temos a = 0
Para Y, temos b = tg α1 = = = 1
Para Z, temos c = tg α2 = = = 2
Como as curvas W e Z são paralelas, apresentam o
mesmo coeficiente angular ∴ d = 2
A equação da velocidade pode ser expressa por
v = k [X]0 . [Y]1 . [Z]2 . [W]2
A ordem global da reação será:
0 + 1 + 2 + 2 = 5
0,4
––––
0,2
(– 0,6) – (– 1,0)
–––––––––––––
(– 0,8) – (1,0)
0,2
––––
0,2
(– 0,8) – (– 1,0)
–––––––––––––
(– 0,8) – (1,0)
IIIITTTTAAAA ---- ((((4444ºººº DDDDiiiiaaaa)))) DDDDeeeezzzzeeeemmmmbbbbrrrroooo////2222000000005555
OOOOBBBBJJJJEEEETTTTIIIIVVVVOOOO
13 EEEE
Considere soluções de SiCl4/CCl4 de frações molares
variáveis, todas a 25° C. Sabendo que a pressão de va-
por do CCl4 a 25° C é igual a 114,9 mmHg, assinale a
opção que mostra o gráfico que melhor representa a
pressão de vapor de CCl4 (PCCl4
) em função da fração
molar de SiCl4 no líquido (X1
SiCl4
) .
IIIITTTTAAAA ---- ((((4444ºººº DDDDiiiiaaaa)))) DDDDeeeezzzzeeeemmmmbbbbrrrroooo////2222000000005555
OOOOBBBBJJJJEEEETTTTIIIIVVVVOOOO
Resolução
Admitindo CCl4 puro (X’SiCl4
= 0), a pressão de vapor do
CCl4 a 25°C vale 114,9mmHg.
Admitindo SiCl4 puro (X’SiCl4
= 1), a pressão de vapor do
CCl4 será 0 (zero).
As alternativas a, b, c são eliminadas.
Admitindo uma solução ideal (os dois líquidos com-
pletamente miscíveis entre si), a pressão de vapor do
CCl4 é dada pela Lei de Raoult:
PCCl4
= P0
CCl4
. XCCl4
em que P0
CCl4
é a pressão de vapor do CCl4 puro na
mesma temperatura e XCCl4
, a fração do CCl4 na mis-
tura.
Como a fração do CCl4 vai diminuindo à medida que a
fração do SiCl4 aumenta, o gráfico PCCl4
/ X’SiCl4
será
uma reta decrescente.
IIIITTTTAAAA ---- ((((4444ºººº DDDDiiiiaaaa)))) DDDDeeeezzzzeeeemmmmbbbbrrrroooo////2222000000005555
OOOOBBBBJJJJEEEETTTTIIIIVVVVOOOO
14 CCCC
Um recipiente fechado, mantido a volume e tempe-
ratura constantes, contém a espécie química X no
estado gasoso a pressão inicial Po. Esta espécie
decompõe-se em Y e Z de acordo com a seguinte equa-
ção química: X(g) → 2Y(g) + 1/2 Z(g). Admita que X, Y e
Z tenham comportamento de gases ideais. Assinale a
opção que apresenta a expressão CORRETA da
pressão (P) no interior do recipiente em função do
andamento da reação, em termos da fração α de
moléculas de X que reagiram.
a) P = [1 + (1/2)α] Po b) P = [1 + (2/2)α] Po
c) P = [1 + (3/2)α] Po d) P = [1+(4/2)α] ]Po
e) P = [1 + (5/2)α] Po
Resolução
A reação ocorre a volume e temperatura constantes, e
os participantes da reação são todos gasosos, logo a
pressão total do sistema depende da quantidade em
mol.
Considere a tabela para a variação da pressão do
sistema.
Seja P a pressão total do sistema em um instante t.
P = P0 (1 – α) + 2αP0 + αP0
P = P0 1 + α΅3–––
2΄
1
–––
2
1
––– αP0
2
2αP0P0(1 – α)instante t
1
––– αP0
2
2αP0αP0reage/forma
00P0início
1X(g) → 2Y(g) + ––– Z(g)
2
IIIITTTTAAAA ---- ((((4444ºººº DDDDiiiiaaaa)))) DDDDeeeezzzzeeeemmmmbbbbrrrroooo////2222000000005555
OOOOBBBBJJJJEEEETTTTIIIIVVVVOOOO
15 AAAA
Um elemento galvânico é constituído pelos eletrodos
abaixo especificados, ligados por uma ponte salina e
conectados a um multímetro de alta impedância.
Eletrodo a: Placa de chumbo metálico mergulhada em
uma solução aquosa 1 mol L–1de nitrato de chumbo.
Eletrodo b: Placa de níquel metálico mergulhada em
uma solução aquosa 1 mol L–1 de sulfato de níquel.
Após estabelecido o equilíbrio químico nas condições-
padrão, determina-se a polaridade dos eletrodos. A
seguir, são adicionadas pequenas porções de KI sólido
ao Eletrodo a, até que ocorra a inversão de polaridade
do elemento galvânico.
Dados eventualmente necessários:
Produto de solubilidade de PbI2: Kps (PbI2) = 8,5 x 10–9
Potenciais de eletrodo em relação ao eletrodo padrão
de hidrogênio nas condições-padrão:
E0
Pb/Pb2+ = – 0,13V ; E0
Ni/Ni2+ = – 0,25V; E0
I–/I2
= 0,53V
Assinale a opção que indica a concentração CORRETA
de KI, em mol L–1, a partir da qual se observa a inversão
de polaridade dos eletrodos nas condições-padrão.
a) 1 x 10–2 b) 1 x 10–3 c) 1 x 10–4
d) 1 x 10–5 e) 1 x 10–6
Resolução
(I) Cálculo do ∆E0 inicial da pilha:
Eletrodo a ᮍ Cátodo: Pb2+(aq) + 2e– →← Pb0(s) – 0,13V
Eletrodo b ᮎ Ânodo: Ni0(s) →← 2e–+ Ni2+(aq) + 0,25V
–––––––––––––––––––––––––––––––––
Equação global Pb2+(aq) + Ni0(s) →← Pb0(s) + Ni2+(aq)
(II)Observa-se a inversão de polaridade dos eletrodos
nas condições-padrão quando E
Pb2+
< – 0,25V
Pela Equação de Nernst, temos:
E = E0 – . log
– 0,25 = – 0,13 – 0,02955 . log .
4,06 = log ∴ [Pb2+] ≅ 10–4
Cálculo da concentração de I–
:
KPS = [Pb2+] [ I –] 2
8,5 . 10–9 = 10–4 [ I –] 2
Logo: [KI] ≅ 10–2mol/L
[ I –] ≅ 10–2mol/L
mol
–––––
L
1
–––––––
[Pb2+]
1
–––––––
[Pb2+]
1
–––––––
[Pb2+]
0,0591
–––––––
2
∆E0 = +0,12V
IIIITTTTAAAA ---- ((((4444ºººº DDDDiiiiaaaa)))) DDDDeeeezzzzeeeemmmmbbbbrrrroooo////2222000000005555
OOOOBBBBJJJJEEEETTTTIIIIVVVVOOOO
16 EEEE
São dadas as semi-equações químicas seguintes e
seus respectivos potenciais elétricos na escala do
eletrodo de hidrogênio nas condições-padrão:
I. Cl2(g)+2e– →
← 2Cl–(aq); E0
I
= +1,358V
II. Fe2+(aq) + 2e– →
← Fe(s) ; E0
II
= –0,447V
III.Fe3+(aq) + 3e– →
← Fe(s) ; E0
III
= –0,037V
IV.Fe3+(aq) + e– →
← Fe2+ (aq) ; E0
IV
= +0,771V
V. O2(g)+4H+(aq)+4e– →
← 2H2O(l) ; E0
V
= + 1,229V
Com base nestas informações, assinale a opção que
contém a afirmação CORRETA, considerando as con-
dições-padrão.
a) A formação de FeCl2 a partir de Fe fundido e Cl2
gasoso apresenta ∆H > 0.
b) Tanto a eletrólise ígnea do FeCl2(s) quanto a do
FeCl3(s), quando realizadas nas mesmas condições
experimentais, produzem as mesmas quantidades
em massa de Fe(s) .
c) Uma solução aquosa de FeCl2 reage com uma
solução aquosa de ácido clorídrico, gerando H2(g).
d) Borbulhando Cl2(g) em uma solução aquosa de Fe2+,
produz-se 1 mol de Fe3+ para cada mol de Cl– em
solução.
e) Fe2+ tende a se oxidar em solução aquosa ácida
quando o meio estiver aerado.
Resolução
Fe2+ tende a oxidar-se em solução aquosa ácida quan-
do o meio estiver aerado.
4Fe2+(aq) →← 4Fe3+(aq) + 4e– E
0
oxid = – 0,771V
O2(g) + 4H+(aq) + 4e– →← 2H2O(l) E
0
red = + 1,229V
––––––––––––––––––––––––––––––––––––––––––––––––––––––––––––
4Fe2+(aq)+O2(g)+4H+(aq) →← 4Fe3+(aq)+2H2O(l) ∆E = 0,458V
IIIITTTTAAAA ---- ((((4444ºººº DDDDiiiiaaaa)))) DDDDeeeezzzzeeeemmmmbbbbrrrroooo////2222000000005555
OOOOBBBBJJJJEEEETTTTIIIIVVVVOOOO
17 CCCC
Duas células (I e II) são montadas como mostrado na
figura. A célula I contém uma solução aquosa 1 mol L–1
em sulfato de prata e duas placas de platina. A célula II
contém uma solução aquosa 1 mol L–1 em sulfato de
cobre e duas placas de cobre. Uma bateria fornece uma
diferença de potencial elétrico de 12 V entre os
eletrodos Ia e IIb, por um certo intervalo de tempo.
Assinale a opção que contém a afirmativa ERRADA em
relação ao sistema descrito.
a) Há formação de O2(g) no eletrodo Ib.
b) Há um aumento da massa do eletrodo Ia.
c) A concentração de íons Ag+ permanece constante na
célula I.
d) Há um aumento de massa do eletrodo lIa.
e) A concentraçao de íons Cu2+ permanece constante
na célula lI.
Resolução
Célula I:
cátodo (Ia): Ag+(aq) + e– → Ag(s)
ânodo (Ib): H2O(l) → 2e– + 1/2 O2(g) + 2H+(aq)
No eletrodo Ib, haverá a formação de O2(g) devido à
oxidação da água.
No eletrodo Ia, haverá a formação de Ag(s) devido à
redução dos íons Ag+(aq), diminuindo a concentração
destes na célula I.
Célula II:
cátodo (IIa): Cu2+(aq) + 2e– → Cu(s)
ânodo (IIb): Cu(s) → 2e– + Cu2+(aq)
No eletrodo IIa, haverá a formação de Cu(s) devido à
redução dos íons Cu2+(aq).
IIIITTTTAAAA ---- ((((4444ºººº DDDDiiiiaaaa)))) DDDDeeeezzzzeeeemmmmbbbbrrrroooo////2222000000005555
OOOOBBBBJJJJEEEETTTTIIIIVVVVOOOO
No eletrodo IIb, ocorrerá a oxidação de Cu(s) a íons
Cu2+(aq), mantendo constante a concentração destes
na célula II.
IIIITTTTAAAA ---- ((((4444ºººº DDDDiiiiaaaa)))) DDDDeeeezzzzeeeemmmmbbbbrrrroooo////2222000000005555
OOOOBBBBJJJJEEEETTTTIIIIVVVVOOOO
18 DDDD
Considere as afirmações abaixo, todas relacionadas a
átomos e íons no estado gasoso:
I. A energia do íon Be2+, no seu estado fundamental, é
igual à energia do átomo de He neutro no seu estado
fundamental.
II. Conhecendo a segunda energia de ionização do
átomo de He neutro, é possível conhecer o valor da
afinidade eletrônica do íon He2+.
III.Conhecendo o valor da afinidade eletrônica e da
primeira energia de ionização do átomo de Li neutro,
é possível conhecer a energia envolvida na emissão
do primeiro estado excitado do átomo de Li neutro
para o seu estado fundamental.
IV.A primeira energia de ionização de íon H
–
é menor do
que a primeira energia de ionização do átomo de H
neutro.
V. O primeiro estado excitado do átomo de He neutro
tem a mesma configuração eletrônica do primeiro
estado excitado do íon Be2+.
Então, das afirmações acima, estão CORRETAS
a) apenas I e III. b) apenas I, II e V.
c) apenas I e IV. d) apenas II, IV e V.
e) apenas III e V.
Resolução
I) Incorreta
A energia do íon Be 2+, que possui 4 protóns e
2 elétrons (maior atração nuclear) é diferente da do
átomo de He neutro, que possui 2 prótons e 2 elé-
trons.
II) Correta
He+ + energia → He2+ + e– (I)
He2+ + e– → He+ + energia (II)
Conhecendo a segunda energia de ionização do
átomo de He (I), é possível conhecer o valor da
afinidade eletrônica do íon He2+ (II).
III)Incorreta
Li + e– → Li – + energia (I)
Li + energia → Li+ + e– (II)
Conhecendo o valor da afinidade eletrônica (I) e da
primeira energia de ionização (II) do átomo de Li
neutro, não é possível conhecer a energia envolvida
na emissão do primeiro estado excitado do átomo de
Li neutro para o seu estado fundamental, no qual o
elétron volta de um subnível mais energético para
um subnível menos energético.
Li (excitado) → Li (fundamental)
IV)Correta
H– + energia → H + e– (I)
H + energia → H+ + e– (II)
A primeira energia de ionização de íon H– (I) é menor
do que a primeira energia de ionização do átomo de
IIIITTTTAAAA ---- ((((4444ºººº DDDDiiiiaaaa)))) DDDDeeeezzzzeeeemmmmbbbbrrrroooo////2222000000005555
OOOOBBBBJJJJEEEETTTTIIIIVVVVOOOO
H neutro (II), pois esta é a segunda energia de
ionização do íon H–.
V) Correta
O primeiro estado excitado do átomo de He neutro
tem a mesma configuração eletrônica do primeiro
estado excitado do íon Be2+, pois as duas espécies
possuem 2 elétrons, e sua configuração não-excitada
é 1s2.
19 BBBB
Uma reação química hipotética é representada pela
seguinte equação: X(g) + Y(g) → 3Z(g). Considere que
esta reação seja realizada em um cilindro provido de um
pistão, de massa desprezível, que se desloca sem
atrito, mantendo-se constantes a pressão em 1 atm e a
temperatura em 25°C. Em relação a este sistema, são
feitas as seguintes afirmações:
I. O calor trocado na reação é igual à variação de
entalpia.
II. O trabalho realizado pelo sistema é igual a zero.
III.A variação da energia interna é menor do que a
variação da entalpia.
IV.A variação da energia interna é igual a zero.
V. A variação da energia livre de Gibbs é igual à variação
de entalpia.
Então, das afIrmações acima, estão CORRETAS
a) apenas I, II e IV. b) apenas I e III.
c) apenas II e V. d) apenas III e IV.
e) apenas III, IV e V.
Resolução
I. Correta.
X(g) + Y(g) → 3Z(g)
2 mol 3 mol
∆H = QP , pressão constante
II. Errada.
τ ≠ 0, τ = ∆n R T, τ = 1 . R T
III.Correta.
∆H = ∆U + τ ∴ ∆H = ∆U + R T
IV.Errada.
∆U ≠ 0, ∆U = ∆H – R T
Observação – Quando não ocorre reação quími-
ca, se a temperatura permanecer constante, a varia-
ção de energia interna é nula. Não é o nosso caso.
V. Errada.
∆G = ∆H – T ∆S
IIIITTTTAAAA ---- ((((4444ºººº DDDDiiiiaaaa)))) DDDDeeeezzzzeeeemmmmbbbbrrrroooo////2222000000005555
OOOOBBBBJJJJEEEETTTTIIIIVVVVOOOO
20 EEEE
A figura apresenta cinco curvas (I, lI, III, IV e V) da
concentração de uma espécie X em função do tempo.
Considerando uma reação química hipotética re-
presentada pela equação X(g) → Y(g), assinale a opção
CORRETA que indica a curva correspondente a uma
reação química que obedece a uma lei de velocidade de
segunda ordem em relação à espécie X.
a) Curva I b) Curva II c) Curva III
d) Curva IV e) Curva V
Resolução
Para uma reação de segunda ordem, a equação de
velocidade é dada pela expressão:
X(g) → Y(g)
v= k [X] 2
Escrevendo a velocidade em função da concentração e
do tempo, temos:
v = = k [X]2
= k d t
Integrando, temos:
= kt
∴ [X] . t = (equação de uma hipérbole)
Dentre as curvas fornecidas, a hipérbole corresponde à
curva V.
Confirmando com os valores fornecidos no gráfico:
1
–––
k
1
–––
[X]
d [X]
– –––––
[X]2
– d [X]
––––––
dt
IIIITTTTAAAA ---- ((((4444ºººº DDDDiiiiaaaa)))) DDDDeeeezzzzeeeemmmmbbbbrrrroooo////2222000000005555
OOOOBBBBJJJJEEEETTTTIIIIVVVVOOOO
Ponto A
[X] . t = 5 mmol L– 1 . 2 s = 10 . mmol . L–1 . s
Ponto B
[X] . t = 4 mmol L– 1 . 2,5 s = 10 mmol . L– 1 . s
Ponto C
[X] . t = 2 mmol L– 1 . 5 s = 10 mmol . L– 1 . s
As questões dissertativas, numeradas de 21 a 30,
devem ser resolvidas e respondidas no caderno de
soluções.
21Considere as informações abaixo:
a) PbCrO4(s) é um sólido amarelo que é solúvel em
água quente.
b) AgCl(s) forma um cátion de prata solúvel em solução
aquosa de NH3.
c) O sólido branco PbCl2(s) é solúvel em água quente,
mas os sólidos brancos AgCl(s) e Hg2Cl2(s) não o
são.
d) Uma solução aquosa contendo o cátion de prata do
item b), quando acidulada com HCl, forma o sólido
AgCl(s).
e) Hg2Cl2(s) forma uma mistura insolúvel contendo
Hg(l), que tem cor prata, e HgNH2Cl(s), que é preto,
em solução aquosa de NH3.
Uma amostra sólida consiste em uma mistura de
cloretos de Ag+, Hg2
2+
e Pb2+. Apresente uma seqüên-
cia de testes experimentais para identificar os íons Ag+,
Hg2
2+
e Pb2+ presentes nesta amostra.
Resolução
Considerando a amostra sólida formada pelos sais
AgCl, Hg2Cl2 e PbCl2, podemos solubilizar o PbCl2
adicionando água quente.
Filtrando o sistema, teremos uma solução contendo
cloreto de Pb2+ solúvel (em água quente) que poderá
ser identificado adicionando uma solução contendo
íons CrO4
2–
.
Pb2+(aq) + CrO4
2–
(aq) → PbCrO4(s)
↓amarelo
Ao resíduo sólido retido no filtro (Hg2Cl2 + AgCl), acres-
centamos solução aquosa de amônia que irá solubilizar
o AgCl(s)(reação de complexação):
AgCl(s) + 2NH3(aq) → Ag(NH3)2
+
(aq) + Cl–(aq)
Filtrando esse novo sistema e adicionando HCl ao
filtrado (excesso de Cl–), haverá precipitação de AgCl,
de cor branca.
Ag(NH3)2
+
(aq) + Cl
–
(aq) → AgCl(s)
↓+ 2NH3(aq)
ppt
No resíduo da última filtração, observamos mercúrio
líquido de cor prata e HgNH2Cl(s) preto.
IIIITTTTAAAA ---- ((((4444ºººº DDDDiiiiaaaa)))) DDDDeeeezzzzeeeemmmmbbbbrrrroooo////2222000000005555
OOOOBBBBJJJJEEEETTTTIIIIVVVVOOOO
22Calcule o valor do potencial elétrico na escala do ele-
trodo de hidrogênio nas condições-padrão (E0) da semi-
equação química CuI(s) + e–(CM) ←
→ Cu(s) + I –(aq).
Dados eventualmente necessários: Produto de solu-
bilidade do CuI(s): Kps (CuI) = 1,0 x 10–12
Semi-equações químicas e seus respectivos potenciais
elétricos na escala do eletrodo de hidrogênio nas
condições-padrão (E0):
I. Cu2+(aq) + e– (CM) ←
→ Cu+(aq) ; E
0
I = 0,15 V
II. Cu2+(aq) + 2e–(CM) ←
→ Cu(s) ; E
0
II =0,34V
III. Cu+(aq) + e– (CM) ←
→ Cu(s) ; E
0
III = 0,52 V
IV. I2(s) + 2e– (CM) ←
→ 2I–(aq) ; E
0
IV = 0,54 V
Resolução
A semi-equação química fornecida:
CuI(s) + e–(CM) →← Cu(s) + I–
(aq) E = ?
redução
representa a redução do íon Cu+ no composto CuI.
O dado número III fornece a semi-equação:
Cu+(aq) + e–(CM) →← Cu(s) E0
III = 0,52V
Como a equação fornecida apresenta o íon Cu+
precipitado na forma de CuI, calcula-se a concentração
final de Cu+ por meio do PS.
CuI(s) →← Cu+(aq) + I–
(aq)
x x
PS = [Cu+] . [I–
]
}x = 1,0 . 10–6
mol/L
1,0 . 10–12
= x2
Cálculo do E:
Pela Equação de Nernst, temos:
E = E0
III
– . log
E = 0,52 – 0,0591 . log
E = + 0,52 – 0,0591 . (6)
E = + 0,52 – 0,3546 = 0,1654
E = 0,1654V
1
––––––
10–6
1
––––––
[Cu+]
0,0591
––––––––
1
01+
IIIITTTTAAAA ---- ((((4444ºººº DDDDiiiiaaaa)))) DDDDeeeezzzzeeeemmmmbbbbrrrroooo////2222000000005555
OOOOBBBBJJJJEEEETTTTIIIIVVVVOOOO
23Esboce graficamente o diagrama de fases (pressão
versus temperatura) da água pura (linhas cheias). Neste
mesmo gráfico, esboce o diagrama de fases de uma
solução aquosa 1 moI kg–1 em etilenoglicol (linhas
tracejadas).
Resolução
IIIITTTTAAAA ---- ((((4444ºººº DDDDiiiiaaaa)))) DDDDeeeezzzzeeeemmmmbbbbrrrroooo////2222000000005555
OOOOBBBBJJJJEEEETTTTIIIIVVVVOOOO
24Uma reação química genérica pode ser representada
pela seguinte equação: A(s) ←
→ B(s) + C(g). Sabe-se
que, na temperatura Teq, esta reação atinge o equilíbrio
químico, no qual a pressão parcial de C é dada por PC,eq
.
Quatro recipientes fechados (I, II, III e IV), mantidos na
temperatura Teq, contêm as misturas de substâncias e
as condições experimentais especificadas abaixo:
I. A(s) + C(g) ; PC,I < PC,eq
ll. A(s) + B(s) ; PC,II = 0
III.A(s) + C(g) ; PC,III >>> Pc,eq
IV.B(s) + C(g) ; PC,IV > PC,eq
Para cada um dos recipientes, o equilíbrio químico ci-
tado pode ser atingido? Justifique suas respostas.
Resolução
Como a pressão de C no re-
cipiente I é menor que a pres-
são de C no equilíbrio, a
substância A irá transformar-se
em B e C até que a pressão de
C seja PC,eq (o equilíbrio pode
ser atingido).
Irá ocorrer a reação,
A(s) →← B (s) + C(g)
A substância A (sólido) irá
produzir mais B (sólido) e C
(gás) até que a pressão de C
seja PC,eq (o equilíbrio pode ser
atingido).
Para que o equilíbrio fosse
atingido, a pressão de C (PC,III)
deveria diminuir, o que é im-
possível por não haver a
substância B (sólida) no reci-
piente.
Como a pressão de C no re-
cipiente é maior que a sua
pressão no equilíbrio, C deverá
reagir com B sólido, produzindo
a substância A (sólida) e o
equilíbrio pode ser atingido.
Obs.: nas experiências I, II e IV, admite-se haver rea-
gente em quantidade suficiente para estabelecer o
equilíbrio.
IIIITTTTAAAA ---- ((((4444ºººº DDDDiiiiaaaa)))) DDDDeeeezzzzeeeemmmmbbbbrrrroooo////2222000000005555
OOOOBBBBJJJJEEEETTTTIIIIVVVVOOOO
25Uma substância A apresenta as seguintes proprie-
dades:
Temperatura de fusão a 1 atm = – 20 °C
Temperatura de ebulição a 1 atm = 85 °C
Variação de entalpia de fusão = 180 J g–1
Variação de entalpia de vaporização = 500 J g– 1
Calor específico de A(s) = 1,0J g–1 °C–1
Calor específico de A(l)= 2,5J g–1 °C–1
Calor específico de A(g) = 0,5J g–1 °C–1
À pressão de 1 atm, uma amostra de 25g de substância
A é aquecida de – 40°C até 100°C, a uma velocidade
constante de 450 J min–1. Considere que todo calor
fornecido é absorvido pela amostra. Construa o gráfico
de temperatura (°C) versus tempo (min) para todo o
processo de aquecimento considerado, indicando
claramente as coordenadas dos pontos iniciais e finais
de cada etapa do processo. Mostre os cálculos
necessários.
Resolução
I) Aquecimento de – 40°C a – 20°C:
Q1 = m c ∆θ
Q1 = 25 . 1 . (20)
Q1 = 500J
Assim:
∆t1 = = = 1,1 min
Em que Pot é potência constante com que o calor
é fornecido à amostra.
II) Fusão da substância A:
Q2 = m LF
Em que LF é a variação de entalpia de fusão.
Q2 = 25 . 180
Q2 = 4500J
∴ ∆t2 = = = 10min
III) Aquecimento da substância no estado líquido:
Q3 = m c ∆θ
Q3 = 25 . 2,5 . (85 – (– 20))
Q3 = 6562,5J
∴ ∆t3 = = = 14,6min
IV) Vaporização da substância A:
Q4 = m LV
Em que LV é a variação da entalpia de vaporização.
Q4 = 25 . 500
6562,5
––––––––
450
Q3
––––
Pot
4500
–––––
450
Q2
––––
Pot
500
––––
450
Q1
––––
Pot
IIIITTTTAAAA ---- ((((4444ºººº DDDDiiiiaaaa)))) DDDDeeeezzzzeeeemmmmbbbbrrrroooo////2222000000005555
OOOOBBBBJJJJEEEETTTTIIIIVVVVOOOO
Q4 = 12500J
∴ ∆t4 = = = 27,8min
V) Aquecimento do vapor:
Q5 = m c ∆θ
Q5 = 25 . 0,5 (15)
Q5 = 187,5J
∴ ∆t5 = = ≅ 0,42min
Dessa maneira, o gráfico pedido é:
Nota: O gráfico não está em escala.
187,5
––––––––
450
Q5
––––
Pot
12500
––––––––
450
Q4
––––
Pot
IIIITTTTAAAA ---- ((((4444ºººº DDDDiiiiaaaa)))) DDDDeeeezzzzeeeemmmmbbbbrrrroooo////2222000000005555
OOOOBBBBJJJJEEEETTTTIIIIVVVVOOOO
26Para cada um dos processos listados abaixo, indique se
a variação de entropia será maior, menor ou igual a
zero.
Justifique suas respostas.
a) N2(g,1 atm, T = 300 K) → N2(g,0,1 atm, T = 300 K)
b) C (grafite) → C(diamante)
c) solução supersaturada → solução saturada
d) sólido amorfo → sólido cristalino
e) N2(g) → N2 (g, adsorvido em sílica)
Resolução
A entropia é uma grandeza termodinâmica que mede o
grau de desordem de um sistema.
maior desordem → maior entropia
a) N2(g, 1 atm, T = 300K) → N2 (g, 0,1 atm, T = 300K)
S1 S2
S2 > S1 : ∆S > 0, pois houve expansão de volume
(diminuição de pressão), aumentando a desordem.
b) C(grafita) → C(diamante)
S1 S2
menos mais
organizado organizado
S1 > S2 : ∆S < 0
c) Solução supersaturada → solução saturada
S1 S2
S1 > S2 : ∆S < 0, parte das partículas que estavam na
solução supersaturada cristalizaram, portanto, dimi-
nuindo a entropia total do sistema.
d) sólido amorfo → sólido cristalino
partículas em partículas em
desordem ordem
S1 S2
S1 > S2 : ∆S < 0
e) N2(g) → N2 (g, adsorvido em sílica)
S1 S2
S1 > S2 : ∆S < 0
IIIITTTTAAAA ---- ((((4444ºººº DDDDiiiiaaaa)))) DDDDeeeezzzzeeeemmmmbbbbrrrroooo////2222000000005555
OOOOBBBBJJJJEEEETTTTIIIIVVVVOOOO
27A equação química hipotética A → D ocorre por um me-
canismo que envolve as três reações unimoleculares
abaixo (I, II e III). Nestas reações, ∆Hi representa as
variações de entalpia, e Eai, as energias de ativação.
I. A → B; rápida, ∆HI, EaI
II. B → C; lenta, ∆HII, EaII
III.C → D; rápida, ∆HIII, EaIII
Trace a curva referente à energia potencial em função
do caminho da reação A → D, admitindo que a reação
global A → D seja exotérmica e considerando que:
∆HII >∆HI > 0; EaI <EaIll.
Resolução
IIIITTTTAAAA ---- ((((4444ºººº DDDDiiiiaaaa)))) DDDDeeeezzzzeeeemmmmbbbbrrrroooo////2222000000005555
OOOOBBBBJJJJEEEETTTTIIIIVVVVOOOO
28São realizadas reações químicas do acetileno com áci-
do clorídrico, ácido cianídrico, ácido acético e cloro, nas
proporções estequiométricas de 1:1.
a) Mostre as equações químicas que representam cada
uma das reações químicas especificadas.
b) Indique quais dos produtos formados podem ser uti-
lizados como monômeros na síntese de polímeros.
c) Dê os nomes dos polímeros que podem ser
formados a partir dos monômeros indicados no item
b).
Resolução
H Cl
| |
a) H — C ≡ C — H + HCl → H — C = C — H
H CN
| |
H — C ≡ C — H + HCN → H — C = C — H
O
H — C ≡ C — H + C — CH3 →
HO
O
C — CH3
H O
| |
→ H — C = C — H Cl Cl
| |
H — C ≡ C — H + Cl2 → H — C = C — H
b) Todos os produtos são usados como monômeros.
c) poli(cloreto de vinila) ou PVC
poli(cianeto de vinila) ou poliacrilonitrilo ou orlon
poli(acetato de vinila) ou PVA
poli(1,2-dicloroetileno)
IIIITTTTAAAA ---- ((((4444ºººº DDDDiiiiaaaa)))) DDDDeeeezzzzeeeemmmmbbbbrrrroooo////2222000000005555
OOOOBBBBJJJJEEEETTTTIIIIVVVVOOOO
29São descritos, a seguir, dois experimentos e respec-
tivas observações envolvendo ossos limpos e secos
provenientes de uma ave.
I. Um osso foi imerso em uma solução aquosa 10% (v/v)
em ácido fórmico. Após certo tempo, observou-se
que o mesmo havia se tornado flexível.
II. Um outro osso foi colocado em uma cápsula de
porcelana e submetido a aquecimento em uma cha-
ma de bico de Bunsen. Após um longo período de
tempo, observou-se que o mesmo se tornou frágil e
quebradiço.
a) Explique as observações descritas nos dois expe-
rimentos.
b) Baseando-se nas observações acima, preveja o que
acontecerá se um terceiro osso limpo e seco for
imerso em uma solução aquosa 1 mg L–1 em fluore-
to de sódio e, a seguir, em uma solução aquosa a
10% (v/v) em ácido fórmico. Justifique a sua res-
posta.
Resolução
Experiência I:
a) Com a retirada do cálcio, pela imersão em ácido, o
osso se torna flexível devido à permanência de
substância orgânica intersticial (responsável pela
flexibilidade).
O osso é constituído por um sal iônico, a hidro-
xiapatita, de fórmula Ca10(PO4)6 (OH)2 e substân-
cias orgânicas
O
Reação:Ca10(PO4)6(OH)2(s) + 2 HC (aq) →
OH
→ Ca10(PO4)6(HCOO)2(aq) + 2H2O(l)
Experiência II:
Há eliminação, pelo aquecimento, do material orgânico
intersticial – osseína – , restando apenas o sal inor-
gânico, Ca10(PO4)6(OH)2, bem como a retirada da H2O,
por vaporização, reduzindo a flexibilidade do osso, tor-
nando-o mais frágil e quebradiço.
b) O terceiro osso, imerso em solução de fluoreto de
sódio, é fortalecido, impedindo que posteriormente
o sal de flúor formado seja dissolvido com o ácido.
Ca10(PO4)6(OH)2(s) + 2NaF(aq) →
→ Ca10(PO4)6F2(s) + 2NaOH(aq)
resistente ao ácido
IIIITTTTAAAA ---- ((((4444ºººº DDDDiiiiaaaa)))) DDDDeeeezzzzeeeemmmmbbbbrrrroooo////2222000000005555
OOOOBBBBJJJJEEEETTTTIIIIVVVVOOOO
30Considere as seguintes espécies no estado gasoso:
BF3, SnF
–
3, BrF3, KrF4 e BrF5. Para cada uma delas, qual
é a hibridização do átomo central e qual o nome da
geometria molecular?
Resolução
BF3 24 elétrons = 12 pares de elétrons
3 pares de elétrons no átomo central: hibridização sp2
geometria molecular: plana trigonal
SnF3
–
26 elétrons = 13 pares de elétrons
4 pares de elétrons: hibridização sp3
geometria molecular: piramidal
BrF3 28 elétrons = 14 pares de elétrons
5 pares de elétrons: hibridização dsp3
geometria molecular: forma de T
KrF4 36 elétrons = 18 pares de elétrons
6 pares de elétrons: hibridização d2sp3
geometria molecular: plana quadrada
BrF5 42 elétrons = 21 pares de elétrons
IIIITTTTAAAA ---- ((((4444ºººº DDDDiiiiaaaa)))) DDDDeeeezzzzeeeemmmmbbbbrrrroooo////2222000000005555
OOOOBBBBJJJJEEEETTTTIIIIVVVVOOOO
6 pares de elétrons: hibridização d2sp3
geometria molecular: pirâmide de base quadrada
CCCCOOOOMMMMEEEENNNNTTTTÁÁÁÁRRRRIIIIOOOO EEEE GGGGRRRRÁÁÁÁFFFFIIIICCCCOOOO
Como já é tradicional, a prova de Química do ITA foi
de nível elevadíssimo, com várias questões cujos
assuntos não estão no programa do Ensino Médio. A
distribuição dos assuntos foi totalmente irregular: 20
questões de físico-química, 6 de química inorgânica e 4
de química orgânica.
IIIITTTTAAAA ---- ((((4444ºººº DDDDiiiiaaaa)))) DDDDeeeezzzzeeeemmmmbbbbrrrroooo////2222000000005555

Mais conteúdo relacionado

Mais procurados

Ita2010 4dia
Ita2010 4diaIta2010 4dia
Ita2010 4diacavip
 
Balanceamentodeequacoes
BalanceamentodeequacoesBalanceamentodeequacoes
BalanceamentodeequacoesDenis Weverton
 
Apostila de cursinho estequiometria e gases (2)
Apostila de cursinho estequiometria e gases (2)Apostila de cursinho estequiometria e gases (2)
Apostila de cursinho estequiometria e gases (2)Leonidas3112
 
Lista 5 estequiometria
Lista 5   estequiometriaLista 5   estequiometria
Lista 5 estequiometriaColegio CMC
 
Lista 25 cálculo estequiométrico
Lista 25   cálculo estequiométricoLista 25   cálculo estequiométrico
Lista 25 cálculo estequiométricoColegio CMC
 
Reacoes quimicas e estequiometria
Reacoes quimicas e estequiometriaReacoes quimicas e estequiometria
Reacoes quimicas e estequiometriaEstude Mais
 
4 lista de exercícios nox
4 lista de exercícios nox4 lista de exercícios nox
4 lista de exercícios noxEdney Marques
 
Cadernoexerciciosquiminorg2ano
Cadernoexerciciosquiminorg2anoCadernoexerciciosquiminorg2ano
Cadernoexerciciosquiminorg2anoMonique Ayala
 
44096407 calculos-estequiometricos-profâº-agamenon-roberto
44096407 calculos-estequiometricos-profâº-agamenon-roberto44096407 calculos-estequiometricos-profâº-agamenon-roberto
44096407 calculos-estequiometricos-profâº-agamenon-robertodebiereis
 
Lista 50 estequiometria
Lista 50   estequiometriaLista 50   estequiometria
Lista 50 estequiometriaColegio CMC
 
Quimica: Exercicios de MOL
Quimica: Exercicios de MOLQuimica: Exercicios de MOL
Quimica: Exercicios de MOLEstude Mais
 
Gabarito 3o. ano química - 2o. bim
Gabarito 3o. ano   química - 2o. bimGabarito 3o. ano   química - 2o. bim
Gabarito 3o. ano química - 2o. bimstjamesmkt
 
Exercícios Estequiometria
Exercícios EstequiometriaExercícios Estequiometria
Exercícios Estequiometriaiqscquimica
 
Exercícios físico- química II-equilíbrio químico
Exercícios  físico- química II-equilíbrio químicoExercícios  físico- química II-equilíbrio químico
Exercícios físico- química II-equilíbrio químicoRenata Martins
 
Ita2013 4 dia
Ita2013 4 diaIta2013 4 dia
Ita2013 4 diacavip
 
Exe massa atomica e molecular, transformações massa mol-moléculas
Exe massa atomica e molecular, transformações massa  mol-moléculasExe massa atomica e molecular, transformações massa  mol-moléculas
Exe massa atomica e molecular, transformações massa mol-moléculassamuelr81
 

Mais procurados (20)

Calculo
CalculoCalculo
Calculo
 
Ita2010 4dia
Ita2010 4diaIta2010 4dia
Ita2010 4dia
 
126 quimica ita 2010
126 quimica ita 2010126 quimica ita 2010
126 quimica ita 2010
 
Balanceamentodeequacoes
BalanceamentodeequacoesBalanceamentodeequacoes
Balanceamentodeequacoes
 
Apostila de cursinho estequiometria e gases (2)
Apostila de cursinho estequiometria e gases (2)Apostila de cursinho estequiometria e gases (2)
Apostila de cursinho estequiometria e gases (2)
 
Lista 5 estequiometria
Lista 5   estequiometriaLista 5   estequiometria
Lista 5 estequiometria
 
Lista 25 cálculo estequiométrico
Lista 25   cálculo estequiométricoLista 25   cálculo estequiométrico
Lista 25 cálculo estequiométrico
 
Reacoes quimicas e estequiometria
Reacoes quimicas e estequiometriaReacoes quimicas e estequiometria
Reacoes quimicas e estequiometria
 
4 lista de exercícios nox
4 lista de exercícios nox4 lista de exercícios nox
4 lista de exercícios nox
 
Cadernoexerciciosquiminorg2ano
Cadernoexerciciosquiminorg2anoCadernoexerciciosquiminorg2ano
Cadernoexerciciosquiminorg2ano
 
44096407 calculos-estequiometricos-profâº-agamenon-roberto
44096407 calculos-estequiometricos-profâº-agamenon-roberto44096407 calculos-estequiometricos-profâº-agamenon-roberto
44096407 calculos-estequiometricos-profâº-agamenon-roberto
 
Lista 50 estequiometria
Lista 50   estequiometriaLista 50   estequiometria
Lista 50 estequiometria
 
Quimica: Exercicios de MOL
Quimica: Exercicios de MOLQuimica: Exercicios de MOL
Quimica: Exercicios de MOL
 
Gabarito 3o. ano química - 2o. bim
Gabarito 3o. ano   química - 2o. bimGabarito 3o. ano   química - 2o. bim
Gabarito 3o. ano química - 2o. bim
 
Oxirredução
OxirreduçãoOxirredução
Oxirredução
 
Exercícios Estequiometria
Exercícios EstequiometriaExercícios Estequiometria
Exercícios Estequiometria
 
Exercícios físico- química II-equilíbrio químico
Exercícios  físico- química II-equilíbrio químicoExercícios  físico- química II-equilíbrio químico
Exercícios físico- química II-equilíbrio químico
 
105
105105
105
 
Ita2013 4 dia
Ita2013 4 diaIta2013 4 dia
Ita2013 4 dia
 
Exe massa atomica e molecular, transformações massa mol-moléculas
Exe massa atomica e molecular, transformações massa  mol-moléculasExe massa atomica e molecular, transformações massa  mol-moléculas
Exe massa atomica e molecular, transformações massa mol-moléculas
 

Destaque

Jbit Medpro Compensation Plan
Jbit Medpro Compensation PlanJbit Medpro Compensation Plan
Jbit Medpro Compensation PlanJbit Affiliate
 
Paradigmas tecnoeconomicos
Paradigmas tecnoeconomicosParadigmas tecnoeconomicos
Paradigmas tecnoeconomicosrocio melendez
 
Apresentação equipe santaella
Apresentação equipe santaellaApresentação equipe santaella
Apresentação equipe santaellaEduardo Junqueira
 
Nereida diapocitivas hobby
Nereida diapocitivas hobbyNereida diapocitivas hobby
Nereida diapocitivas hobbynemumar
 
Economia programa estabilidady pnr
Economia programa estabilidady pnrEconomia programa estabilidady pnr
Economia programa estabilidady pnrManfredNolte
 
PRESENTACIÓN PROYECTO DE AULA.
PRESENTACIÓN PROYECTO DE AULA.PRESENTACIÓN PROYECTO DE AULA.
PRESENTACIÓN PROYECTO DE AULA.jufevibla
 
Advanced motion controls dpcants 060b080
Advanced motion controls dpcants 060b080Advanced motion controls dpcants 060b080
Advanced motion controls dpcants 060b080Electromate
 
A crise de governança e a revolução do conhecimento
A crise de governança e a revolução do conhecimentoA crise de governança e a revolução do conhecimento
A crise de governança e a revolução do conhecimentoMarcos CAVALCANTI
 
Automated Testing Talk from Meet Magento New York 2014
Automated Testing Talk from Meet Magento New York 2014Automated Testing Talk from Meet Magento New York 2014
Automated Testing Talk from Meet Magento New York 2014Joshua Warren
 
Jbit Medpro Review - Lower Limb Purdue Biomechanical Analysis
Jbit Medpro Review - Lower Limb Purdue Biomechanical AnalysisJbit Medpro Review - Lower Limb Purdue Biomechanical Analysis
Jbit Medpro Review - Lower Limb Purdue Biomechanical AnalysisJbit Affiliate
 
Un paseo en Star Wars Celebration Anaheim 2015
Un paseo en Star Wars Celebration Anaheim 2015Un paseo en Star Wars Celebration Anaheim 2015
Un paseo en Star Wars Celebration Anaheim 2015The Force Perú
 
Ita2006 2dia
Ita2006 2diaIta2006 2dia
Ita2006 2diacavip
 

Destaque (20)

Twiterr
TwiterrTwiterr
Twiterr
 
Jbit Medpro Compensation Plan
Jbit Medpro Compensation PlanJbit Medpro Compensation Plan
Jbit Medpro Compensation Plan
 
Informática
InformáticaInformática
Informática
 
Paradigmas tecnoeconomicos
Paradigmas tecnoeconomicosParadigmas tecnoeconomicos
Paradigmas tecnoeconomicos
 
Web 2.0 / Social Media
Web 2.0 / Social MediaWeb 2.0 / Social Media
Web 2.0 / Social Media
 
Apresentação equipe santaella
Apresentação equipe santaellaApresentação equipe santaella
Apresentação equipe santaella
 
Nereida diapocitivas hobby
Nereida diapocitivas hobbyNereida diapocitivas hobby
Nereida diapocitivas hobby
 
Economia programa estabilidady pnr
Economia programa estabilidady pnrEconomia programa estabilidady pnr
Economia programa estabilidady pnr
 
PRESENTACIÓN PROYECTO DE AULA.
PRESENTACIÓN PROYECTO DE AULA.PRESENTACIÓN PROYECTO DE AULA.
PRESENTACIÓN PROYECTO DE AULA.
 
Longitudes
Longitudes Longitudes
Longitudes
 
Advanced motion controls dpcants 060b080
Advanced motion controls dpcants 060b080Advanced motion controls dpcants 060b080
Advanced motion controls dpcants 060b080
 
Ciclo menstrual
Ciclo menstrualCiclo menstrual
Ciclo menstrual
 
A crise de governança e a revolução do conhecimento
A crise de governança e a revolução do conhecimentoA crise de governança e a revolução do conhecimento
A crise de governança e a revolução do conhecimento
 
Dhtic proyecto
Dhtic proyectoDhtic proyecto
Dhtic proyecto
 
Automated Testing Talk from Meet Magento New York 2014
Automated Testing Talk from Meet Magento New York 2014Automated Testing Talk from Meet Magento New York 2014
Automated Testing Talk from Meet Magento New York 2014
 
Jbit Medpro Review - Lower Limb Purdue Biomechanical Analysis
Jbit Medpro Review - Lower Limb Purdue Biomechanical AnalysisJbit Medpro Review - Lower Limb Purdue Biomechanical Analysis
Jbit Medpro Review - Lower Limb Purdue Biomechanical Analysis
 
Apostila de dev
Apostila de devApostila de dev
Apostila de dev
 
Un paseo en Star Wars Celebration Anaheim 2015
Un paseo en Star Wars Celebration Anaheim 2015Un paseo en Star Wars Celebration Anaheim 2015
Un paseo en Star Wars Celebration Anaheim 2015
 
El gatt
El gattEl gatt
El gatt
 
Ita2006 2dia
Ita2006 2diaIta2006 2dia
Ita2006 2dia
 

Semelhante a Ita 2006 4º dia

Gabarito SAS: L2. qui1. aula 07
Gabarito SAS: L2. qui1. aula 07Gabarito SAS: L2. qui1. aula 07
Gabarito SAS: L2. qui1. aula 07Hebertty Dantas
 
Ita2007 4dia
Ita2007 4diaIta2007 4dia
Ita2007 4diacavip
 
Ita2011 4dia
Ita2011 4diaIta2011 4dia
Ita2011 4diacavip
 
Recuperação anual 2 moderna
Recuperação anual 2 modernaRecuperação anual 2 moderna
Recuperação anual 2 modernaAnalynne Almeida
 
Ita2009 4dia
Ita2009 4diaIta2009 4dia
Ita2009 4diacavip
 
Estequiometria conceitos e aplicações
Estequiometria   conceitos e aplicaçõesEstequiometria   conceitos e aplicações
Estequiometria conceitos e aplicaçõesProfª Alda Ernestina
 
estequiometria-conceitoseaplicaes-140415075418-phpapp02.pdf
estequiometria-conceitoseaplicaes-140415075418-phpapp02.pdfestequiometria-conceitoseaplicaes-140415075418-phpapp02.pdf
estequiometria-conceitoseaplicaes-140415075418-phpapp02.pdfRicardoBrunoFelix
 
2°em química rec final_exercícios
2°em química rec final_exercícios2°em química rec final_exercícios
2°em química rec final_exercíciosAdriana Ignácio
 
56553431 1ª-aula-extra-quimica-2011-professor
56553431 1ª-aula-extra-quimica-2011-professor56553431 1ª-aula-extra-quimica-2011-professor
56553431 1ª-aula-extra-quimica-2011-professorConcurseiro De Plantão
 
Ita2012 4dia
Ita2012 4diaIta2012 4dia
Ita2012 4diacavip
 
www.AulasDeQuímicaApoio.com - Química - Cálculo Estequimétrico (parte 2)
www.AulasDeQuímicaApoio.com  - Química -  Cálculo Estequimétrico (parte 2)www.AulasDeQuímicaApoio.com  - Química -  Cálculo Estequimétrico (parte 2)
www.AulasDeQuímicaApoio.com - Química - Cálculo Estequimétrico (parte 2)Aulas de Química Apoio
 
Lista de exercícios VI Termoquímica
Lista de exercícios VI TermoquímicaLista de exercícios VI Termoquímica
Lista de exercícios VI TermoquímicaCarlos Priante
 
Ita2008 4dia
Ita2008 4diaIta2008 4dia
Ita2008 4diacavip
 
Balanceamento de equações químicas.ppt
Balanceamento de equações químicas.pptBalanceamento de equações químicas.ppt
Balanceamento de equações químicas.pptLeviGonalvesdosSanto1
 

Semelhante a Ita 2006 4º dia (20)

Gabarito SAS: L2. qui1. aula 07
Gabarito SAS: L2. qui1. aula 07Gabarito SAS: L2. qui1. aula 07
Gabarito SAS: L2. qui1. aula 07
 
Ita2007 4dia
Ita2007 4diaIta2007 4dia
Ita2007 4dia
 
Resumos experimentos QG109
Resumos experimentos QG109Resumos experimentos QG109
Resumos experimentos QG109
 
Ita2011 4dia
Ita2011 4diaIta2011 4dia
Ita2011 4dia
 
Ita2011 4dia
Ita2011 4diaIta2011 4dia
Ita2011 4dia
 
Recuperação anual 2 moderna
Recuperação anual 2 modernaRecuperação anual 2 moderna
Recuperação anual 2 moderna
 
Eletroquimica
EletroquimicaEletroquimica
Eletroquimica
 
Ita2009 4dia
Ita2009 4diaIta2009 4dia
Ita2009 4dia
 
Estequiometria conceitos e aplicações
Estequiometria   conceitos e aplicaçõesEstequiometria   conceitos e aplicações
Estequiometria conceitos e aplicações
 
estequiometria-conceitoseaplicaes-140415075418-phpapp02.pdf
estequiometria-conceitoseaplicaes-140415075418-phpapp02.pdfestequiometria-conceitoseaplicaes-140415075418-phpapp02.pdf
estequiometria-conceitoseaplicaes-140415075418-phpapp02.pdf
 
2°em química rec final_exercícios
2°em química rec final_exercícios2°em química rec final_exercícios
2°em química rec final_exercícios
 
56553431 1ª-aula-extra-quimica-2011-professor
56553431 1ª-aula-extra-quimica-2011-professor56553431 1ª-aula-extra-quimica-2011-professor
56553431 1ª-aula-extra-quimica-2011-professor
 
Exercícios
ExercíciosExercícios
Exercícios
 
Ita2012 4dia
Ita2012 4diaIta2012 4dia
Ita2012 4dia
 
www.AulasDeQuímicaApoio.com - Química - Cálculo Estequimétrico (parte 2)
www.AulasDeQuímicaApoio.com  - Química -  Cálculo Estequimétrico (parte 2)www.AulasDeQuímicaApoio.com  - Química -  Cálculo Estequimétrico (parte 2)
www.AulasDeQuímicaApoio.com - Química - Cálculo Estequimétrico (parte 2)
 
Lista de exercícios VI Termoquímica
Lista de exercícios VI TermoquímicaLista de exercícios VI Termoquímica
Lista de exercícios VI Termoquímica
 
Ita2008 4dia
Ita2008 4diaIta2008 4dia
Ita2008 4dia
 
Oxidação
OxidaçãoOxidação
Oxidação
 
Reações químicas
Reações químicasReações químicas
Reações químicas
 
Balanceamento de equações químicas.ppt
Balanceamento de equações químicas.pptBalanceamento de equações químicas.ppt
Balanceamento de equações químicas.ppt
 

Mais de cavip

Sf2n3 2010
Sf2n3 2010Sf2n3 2010
Sf2n3 2010cavip
 
Sf2n2 2010
Sf2n2 2010Sf2n2 2010
Sf2n2 2010cavip
 
Sf2n1 2010
Sf2n1 2010Sf2n1 2010
Sf2n1 2010cavip
 
Pf2n3 2010
Pf2n3 2010Pf2n3 2010
Pf2n3 2010cavip
 
Pf2n2 2010
Pf2n2 2010Pf2n2 2010
Pf2n2 2010cavip
 
Pf2n1 2010
Pf2n1 2010Pf2n1 2010
Pf2n1 2010cavip
 
Sf2n3 2011
Sf2n3 2011Sf2n3 2011
Sf2n3 2011cavip
 
Sf2n2 2011
Sf2n2 2011Sf2n2 2011
Sf2n2 2011cavip
 
Sf2n1 2011
Sf2n1 2011Sf2n1 2011
Sf2n1 2011cavip
 
Pf2n3 2011
Pf2n3 2011Pf2n3 2011
Pf2n3 2011cavip
 
Pf2n2 2011
Pf2n2 2011Pf2n2 2011
Pf2n2 2011cavip
 
Pf2n1 2011
Pf2n1 2011Pf2n1 2011
Pf2n1 2011cavip
 
Pf2n3 2012
Pf2n3 2012Pf2n3 2012
Pf2n3 2012cavip
 
Pf2n2 2012
Pf2n2 2012Pf2n2 2012
Pf2n2 2012cavip
 
Pf2n1 2012
Pf2n1 2012Pf2n1 2012
Pf2n1 2012cavip
 
Pf1n3 2012
Pf1n3 2012Pf1n3 2012
Pf1n3 2012cavip
 
Pf1n2 2012
Pf1n2 2012Pf1n2 2012
Pf1n2 2012cavip
 
Pf1n1 2012
Pf1n1 2012Pf1n1 2012
Pf1n1 2012cavip
 
Lpp3 910
Lpp3 910Lpp3 910
Lpp3 910cavip
 
Lpp3 801 pec
Lpp3   801 pecLpp3   801 pec
Lpp3 801 peccavip
 

Mais de cavip (20)

Sf2n3 2010
Sf2n3 2010Sf2n3 2010
Sf2n3 2010
 
Sf2n2 2010
Sf2n2 2010Sf2n2 2010
Sf2n2 2010
 
Sf2n1 2010
Sf2n1 2010Sf2n1 2010
Sf2n1 2010
 
Pf2n3 2010
Pf2n3 2010Pf2n3 2010
Pf2n3 2010
 
Pf2n2 2010
Pf2n2 2010Pf2n2 2010
Pf2n2 2010
 
Pf2n1 2010
Pf2n1 2010Pf2n1 2010
Pf2n1 2010
 
Sf2n3 2011
Sf2n3 2011Sf2n3 2011
Sf2n3 2011
 
Sf2n2 2011
Sf2n2 2011Sf2n2 2011
Sf2n2 2011
 
Sf2n1 2011
Sf2n1 2011Sf2n1 2011
Sf2n1 2011
 
Pf2n3 2011
Pf2n3 2011Pf2n3 2011
Pf2n3 2011
 
Pf2n2 2011
Pf2n2 2011Pf2n2 2011
Pf2n2 2011
 
Pf2n1 2011
Pf2n1 2011Pf2n1 2011
Pf2n1 2011
 
Pf2n3 2012
Pf2n3 2012Pf2n3 2012
Pf2n3 2012
 
Pf2n2 2012
Pf2n2 2012Pf2n2 2012
Pf2n2 2012
 
Pf2n1 2012
Pf2n1 2012Pf2n1 2012
Pf2n1 2012
 
Pf1n3 2012
Pf1n3 2012Pf1n3 2012
Pf1n3 2012
 
Pf1n2 2012
Pf1n2 2012Pf1n2 2012
Pf1n2 2012
 
Pf1n1 2012
Pf1n1 2012Pf1n1 2012
Pf1n1 2012
 
Lpp3 910
Lpp3 910Lpp3 910
Lpp3 910
 
Lpp3 801 pec
Lpp3   801 pecLpp3   801 pec
Lpp3 801 pec
 

Ita 2006 4º dia

  • 1. OOOOBBBBJJJJEEEETTTTIIIIVVVVOOOO QQQQUUUUÍÍÍÍMMMMIIIICCCCAAAA CONSTANTES Constante de Avogadro = 6,02 x 1023 mol–1 Constante de Faraday (F) = 9,65 x 104C mol–1 Volume molar de gás ideal = 22,4 L (CNTP) Carga elementar = 1,602 x 10–19 C Constante dos gases (R) = = 8,21 x 10–2 atm L K–1 moI–1 = 8,31 J K–1 moI–1 = = 62,4 mmHg L K–1 mol–1 = 1,98 cal K–1 mol–1 DEFINIÇÕES Condições normais de temperatura e pressão (CNTP): 0°C e 760 mmHg. Condições ambientes: 25°C e 1 atm. Condições-padrão: 25°C, 1 atm, concentração das so- luções: 1 mol L–1 (rigorosamente: atividade unitária das espécies), sólido com estrutura cristalina mais estável nas condições de pressão e temperatura em questão. (s) ou (c) = sólido cristalino; (l) ou (l) = líquido; (g) = gás; (aq) = aquoso; (graf) = grafite; (CM) = circuito metálico; (conc) = concentrado; (ua) = unidades arbitrárias; [A] = concentração da espécie química A em mol L–1. MASSAS MOLARES Massa Molar (g mol–1 ) 1,01 4,00 6,94 9,01 10,81 12,01 14,01 16,00 19,00 22,99 28,09 30,97 32,06 35,45 Número Atômico 1 2 3 4 5 6 7 8 9 11 14 15 16 17 Elemento Químico H He Li Be B C N O F Na Si P S Cl IIIITTTTAAAA ---- ((((4444ºººº DDDDiiiiaaaa)))) DDDDeeeezzzzeeeemmmmbbbbrrrroooo////2222000000005555
  • 2. OOOOBBBBJJJJEEEETTTTIIIIVVVVOOOO As questões de 01 a 20 NÃO devem ser resolvidas no caderno de soluções. Para respondê-Ias, marque a opção escolhida para cada questão na folha de leitura óptica e na reprodução da folha de leitura óptica (que se encontra na última página do caderno de soluções). 1 AAAA Considere uma amostra nas condições ambientes que contém uma mistura racêmica constituída das subs- tâncias dextrógira e levógira do tartarato duplo de sódio e amônio. Assinale a opção que contém o método mais adequado para a separação destas substâncias. a) Catação. b) Filtração. c) Destilação. d) Centrifugação. e) Levigação. Resolução Isômeros ópticos (dextrogiros e levogiros) desviam o plano de vibração da luz polarizada. Uma mistura racêmica é constituída por partes iguais dos isômeros dextrogiro e levogiro. As propriedades físicas desses isômeros são iguais e, portanto, a única maneira de separá-los, dentre as alternativas citadas, seria por catação por formarem cristais diferentes em suas formas enantiomorfas. Esses cristais poderiam ser separados por uma pinça. Os isômeros poderiam também ser separados por processos químicos. Massa Molar (g mol–1 ) 39,10 40,08 52,00 54,94 55,85 58,69 63,55 79,91 83,80 107,87 118,71 126,90 200,59 207,21 Número Atômico 19 20 24 25 26 28 29 35 36 47 50 53 80 82 Elemento Químico K Ca Cr Mn Fe Ni Cu Br Kr Ag Sn I Hg Pb IIIITTTTAAAA ---- ((((4444ºººº DDDDiiiiaaaa)))) DDDDeeeezzzzeeeemmmmbbbbrrrroooo////2222000000005555
  • 3. OOOOBBBBJJJJEEEETTTTIIIIVVVVOOOO 2 DDDD Considere os seguintes óxidos (I, ll, III, IV e V): I. CaO lI. N2O5 III.Na2O IV. P2O5 V. SO3 Assinale a opção que apresenta os óxidos que, quando dissolvidos em água pura, tornam o meio ácido. a) Apenas I e IV b) Apenas I, III e V c) Apenas II e III d) Apenas lI, IV e V e) Apenas III e V Resolução I) CaO + H2O → Ca(OH)2, meio básico II) N2O5 + H2O → 2HNO3, meio ácido III) Na2O + H2O → 2NaOH, meio básico IV) P2O5 + 3H2O → 2H3PO4, meio ácido V) SO3 + H2O → H2SO4, meio ácido 3 BBBB Assinale a opção que apresenta a equação química que representa uma reação envolvendo a uréia [CO(NH2)2] que NÃO ocorre sob aquecimento a 90 °C e pressão de 1 atm. a) CO(NH2)2(s) + 2HNO2(aq) → → 2N2(g) + CO2(g) + 3H2O(l) b) CO(NH2)2(s) → N2(g) + 1/2 O2(g) + CH4(g) c) CO(NH2)2(s) + H2O(l) → 2NH3(g) + CO2(g) d) CO(NH2)2(s) + H2O(l) + 2HCl(aq) → → 2NH4Cl(aq) + CO2(g) e) CO(NH2)2(s) + 2 NaOH (aq) → Na2CO3(aq) + 2NH3(g) Resolução Uréia é fracamente básica e forma sais com os ácidos fortes. É hidrolisada em presença de ácidos, álcalis ou da enzima urease. a) Ocorre. A uréia reage com ácido nitroso liberando gás nitrogênio. b) Não ocorre, pois a uréia é um composto estável. Ela se decompõe produzindo NH3 e CO2 na forma de equilíbrio: 2NH3 + CO2 → ← CO(NH2)2 + H2O c) Ocorre. Hidrolisa formando amônia e gás carbô- nico. H2O CO(NH2)2(s) → 2NH3(g) + CO2(g) d) Ocorre. Hidrolisa em meio ácido formando íon amônio e gás carbônico. H2O CO(NH2)2(s) → NH4 + (aq) + CO2(g) H+ e) Ocorre. Hidrolisa em meio alcalino formando carbonato e amônia. H2O CO(NH2)2(s) → CO3 2– + NH3 OH– IIIITTTTAAAA ---- ((((4444ºººº DDDDiiiiaaaa)))) DDDDeeeezzzzeeeemmmmbbbbrrrroooo////2222000000005555
  • 4. OOOOBBBBJJJJEEEETTTTIIIIVVVVOOOO 4 DDDD São fornecidas as seguintes informações a respeito de titulação ácido-base: a) A figura mostra as curvas de titulação de 30,0 mL de diferentes ácidos (I, ll, llI, IV e V), todos a 0,10 mol L–1, com uma solução aquosa 0,10 moI L–1 em NaOH. b) O indicador fenolftaleína apresenta o intervalo de mudança de cor entre pH 8,0 a 10,0, e o indicador vermelho de metiIa, entre pH 4,0 a 6,0. Considerando estas informações, é CORRETO afirmar que a) o indicador vermelho de metila é mais adequado que a fenolftaleína para ser utilizado na titulação do ácido IV. b) o indicador vermelho de metila é mais adequado que a fenolftaleína para ser utilizado na titulação do ácido V. c) o ácido III é mais forte que o ácido ll. d) os dois indicadores (fenolftaleína e vermelho de metila) são adequados para a titulação do ácido I. e) os dois indicadores (fenolftaleína e vermelho de metila) são adequados para a titulação do ácido llI. Resolução Dentre os cinco ácidos citados, o mais forte é o I, pois para uma mesma concentração (0,10 mol L–1) é o que apresenta menor pH (maior concentração de íons H+): I > II > III > IV > V A neutralização de um ácido forte com uma base forte (NaOH) produz um sal que em solução resulta num meio neutro, com pH próximo de 7 (25°C). Se neutralizarmos um ácido fraco com uma base forte (NaOH), o meio resultante será básico e, nesse caso, devemos utilizar um indicador com pH de viragem aci- ma de 7 (somente a fenolftaleína). Exceção feita ao ácido I, podemos observar que os demais ácidos apresentam ponto de equivalência em meio de pH acima de 7. Para o ácido I, que apresenta pH de equivalência igual a 7, além da fenolftaleína, podemos usar como indicador o vermelho-de-metila. IIIITTTTAAAA ---- ((((4444ºººº DDDDiiiiaaaa)))) DDDDeeeezzzzeeeemmmmbbbbrrrroooo////2222000000005555
  • 5. OOOOBBBBJJJJEEEETTTTIIIIVVVVOOOO 5 CCCC Considere as seguintes afirmações a respeito da varia- ção, em módulo, da entalpia (∆H) e da energia interna (∆U) das reações químicas, respectivamente represen- tadas pelas equações químicas abaixo, cada uma mantida a temperatura e pressão constantes: I. H2O(g) + 1/2O2(g) → H2O2(g) ; |∆HI| > |∆UI| ll. 4NH3(g) + N2(g) → 3N2H4(g) ; |∆HII| < |∆UII| llI. H2(g) + F2(g) → 2HF(g) ; |∆HIII| > |∆UIII| IV.HCl(g) + 2O2(g) → HClO4(l) ; |∆HIV| < |∆UIV| V. CaO(s) + 3C(s) → CO(g) + CaC2(s); |∆HV| > |∆UV| Das afirmações acima, estão CORRETAS a) apenas I, II e V. b) apenas I, III e IV. c) apenas ll, IV e V. d) apenas III e V. e) todas. Resolução A relação entre variação de entalpia (∆H) e da energia interna (∆U) é: ∆H = ∆U + τ onde τ = ∆n R T ∆n = variação da quantidade de mols dos gases I) H2O(g) + O2(g) → H2O2(g) 1,5 mol 1 mol ∆HI = ∆UI – 0,5RT ͉∆UI ͉ > ͉∆HI ͉ II) 4NH3(g) + N2(g) → 3N2H4(g) 5 mol 3 mol ∆HII = ∆UII – 2RT ͉∆UII͉ > ͉∆HII͉ III) H2(g) + F2(g) → 2HF(g) 2 mol 2 mol ͉∆HIII͉ = ͉∆UIII͉ IV) HCl(g) + 2O2(g) → HClO4(l) 3 mol 0 mol de gás ∆HIV = ∆UIV – 3RT ͉∆UIV ͉ > ͉∆HIV ͉ V) CaO(s) + 3C(s) → CO(g) + CaC2(s) 0 mol de gás 1 mol ∆HV = ∆UV + 1RT ͉∆HV ͉ > ͉∆UV ͉ Corretas: II, IV e V 1 ––– 2 IIIITTTTAAAA ---- ((((4444ºººº DDDDiiiiaaaa)))) DDDDeeeezzzzeeeemmmmbbbbrrrroooo////2222000000005555
  • 6. OOOOBBBBJJJJEEEETTTTIIIIVVVVOOOO 6 AAAA Considere as afirmações abaixo, todas relativas à tem- peratura de 25°C, sabendo que os produtos de solubilidade das substâncias hipotéticas XY, XZ e XW são, respectivamente, iguais a 10–8, 10–12 e 10–16, naquela temperatura. I. Adicionando-se 1 x 10–3 mol do ânion W proveniente de um sal solúvel a 100 mL de uma solução aquosa saturada em XY sem corpo de fundo, observa-se a formação de um sólido. ll. Adicionando-se 1 x 10–3 mol do ânion Y proveniente de um sal solúvel a 100 mL de uma solução aquosa saturada em XW sem corpo de fundo, não se observa a formação de sólido. llI. Adicionando-se 1 x 10–3 mol de XZ sólido a 100 mL de uma solução aquosa contendo 1 x 10–3 moI L–1 de um ânion Z proveniente de um sal solúvel, observa- se um aumento da quantidade de sólido. IV.Adicionando-se uma solução aquosa saturada em XZ sem corpo de fundo a uma solução aquosa saturada em XZ sem corpo de fundo, observa-se a formação de um sólido. Das afirmações acima, está(ão) CORRETA(S) a) apenas I e ll. b) apenas I e llI. c) apenas ll. d) apenas III e IV. e) apenas IV. Resolução Equações de dissociação das substâncias hipotéticas: XY(s) →← X + (aq) + Y – (aq) KPS = [X+] [Y –] = 10–8 XZ(s) →← X + (aq) + Z – (aq) KPS = [X+] [Z –] = 10–12 XW(s) →← X + (aq) + W – (aq) KPS = [X+] [W –] = 10–16 I) Correta. Cálculo da concentração de X + na solução XY: KPS = [X+] [Y –] 10 –8 = x . x x = ͙ෆෆෆ10 –8 x = 10–4 ⇒ [X+] = 10 –4 mol/L Cálculo da concentração de W – : [W – ] = = 10 –2 mol/L X+ + W – →← X W KPS XW = [X + ] [W – ] = 10–16 Quando [X+ ] . [W – ] > 10–16 , temos precipitação: [X+ ] . [W – ] = 10–4 . 10–2 = 10–6 > KPS (formação de sólido) II) Correta. Cálculo da concentração de X + na solução XW: KPS = [X+] [W–] 10 –16 = x . x x = ͙ෆෆෆ10 –16 1 . 10–3 mol ––––––––––– 10–1L IIIITTTTAAAA ---- ((((4444ºººº DDDDiiiiaaaa)))) DDDDeeeezzzzeeeemmmmbbbbrrrroooo////2222000000005555
  • 7. OOOOBBBBJJJJEEEETTTTIIIIVVVVOOOO x = 10–8 ⇒ [X+] = 10 –8 mol/L Cálculo da concentração de Y – : [Y – ] = = 10 –2 mol/L X+ + Y – →← X Y KPS XY = [X + ] [Y – ] = 10–8 Quando [X+ ] . [Y – ] > 10–8 , temos precipitação: [X+ ] . [Y – ] = 10–8 . 10–2 = 10–10 < KPS (não temos formação de sólido) III)Falsa. Cálculo da quantidade, em mol, de X+ e Z– no sólido: XZ → X + + Z – 1 . 10 –3 mol 1 . 10 –3 mol 1 . 10 –3 mol Cálculo da quantidade, em mol, de Z – na solução: 1L ––––– 1 . 10–3 mol }a = 0,1 . 10–3 mol 0,1L ––––– a Quantidade total de Z – : 1,1 . 10–3 mol Equação da reação de precipitação: X+ (aq) + Z– (aq) → XZ(s) 1 . 10 –3 mol 1,1 . 10 –3 mol 1 . 10 –3 mol A quantidade de sólido inicial é praticamente igual à quantidade de sólido final, porque a solubilidade é muito pequena (10–6 mol/L). IV)Falsa. Quando se misturam soluções de mesma con- centração, saturadas sem corpo de fundo, de mesmo soluto, a solução resultante apresentará a mesma concentração, portanto a solução não apre- sentará corpo de fundo. 10–3 mol ––––––––– 10–1L IIIITTTTAAAA ---- ((((4444ºººº DDDDiiiiaaaa)))) DDDDeeeezzzzeeeemmmmbbbbrrrroooo////2222000000005555
  • 8. OOOOBBBBJJJJEEEETTTTIIIIVVVVOOOO 7 AAAA O diagrama de fases da água está representado na figura. Os pontos indicados (I, ll, llI, IV e V) referem-se a sistemas contendo uma mesma massa de água líquida pura em equilíbrio com a(s) eventual(ais) fase(s) termodinamicamente estável(eis) em cada situação. Considere, quando for o caso, que os volumes iniciais da fase vapor são iguais. A seguir, mantendo-se as temperaturas de cada sistema constantes, a pressão é reduzida até Pf. Com base nestas informações, assinale a opção que apresenta a relação ERRADA entre os números de moI de vapor de água (n) presentes nos sistemas, quando a pressão é igual a Pf. a) nI < nIII b) nl < nlV c) nIII < nll d) nIII < nV e) nlV < nV Resolução Ponto I ⇒ apenas líquido Ponto II ⇒ equilíbrio sólido, líquido e vapor Ponto III ⇒ apenas líquido Ponto IV ⇒ equilíbrio líquido-vapor Ponto V ⇒ equilíbrio líquido-vapor A quantidade em mol de líquido em todos os sistemas é a mesma e igual a n. No início, a quantidade em mol para o vapor tem a se- guinte relação: n’V > n’IV > n’II , pois TV > TIV > TII Todos os sistemas sofrem uma expansão isotérmica até Pf , na qual todos apresentam apenas a fase de vapor. Assim, no final, tem-se: nI = n nII = n + x + n’II nIII = n nIV = n + n’IV nV = n + n’V a) Falsa, pois nI = nIII b) Correta, pois nI < nIV c) Correta, pois nIII < nII d) Correta, pois nIII < nV e) Correta, pois nIV < nV IIIITTTTAAAA ---- ((((4444ºººº DDDDiiiiaaaa)))) DDDDeeeezzzzeeeemmmmbbbbrrrroooo////2222000000005555
  • 9. OOOOBBBBJJJJEEEETTTTIIIIVVVVOOOO 8 BBBB Considere um calorímetro adiabático e isotérmico, em que a temperatura é mantida rigorosamente constante e igual a 40°C. No interior deste calorímetro é posi- cionado um frasco de reação cujas paredes permitem a completa e imediata troca de calor. O frasco de reação contém 100 g de água pura a 40°C. Realizam-se cinco experimentos, adicionando uma massa m1 de um sal X ao frasco de reação. Após o estabelecimento do equilíbrio termodinâmico, adiciona-se ao mesmo frasco uma massa m2 de um sal Y e mede-se a variação de entalpia de dissolução (∆H). Utilizando estas informações e as curvas de solubilidade apresentadas na figura, excluindo quaisquer condições de metaes- tabilidade, assinale a opção que apresenta a correlação CORRETA entre as condições em que cada expe- rimento foi realizado e o respectivo ∆H. a) Experimento 1: X = KNO3 ; m1 = 60 g; Y = KNO3 ; m2 = 60 g; ∆H > 0. b) Experimento 2: X = NaClO3; m1 = 40 g; Y = NaClO3; m2 = 40 g; ∆H > 0 c) Experimento 3: X = NaCl ; m1 = 10 g; Y = NaCl; m2 = 10 g; ∆H < 0 d) Experimento 4: X = KNO3 ; m1 = 60 g; Y = NaClO3; m2 = 60 g; ∆H = 0 e) Experimento 5: X = KNO3 ; m1 = 60 g; Y = NH4Cl; m2= 60 g; ∆H < 0 Resolução Um frasco contém 100g de água a 40°C. Adiciona-se uma massa m1 de um sal X ao frasco e, depois de estabelecido o equilíbrio, uma massa m2 de sal Y é adicionada. Mede-se o ∆H de dissolução. Experimento 1. Incorreta: A solubilidade de KNO3 é 60g/100g de H2O a 40°C Sendo: X = KNO3 e m1 = 60g → solução saturada ∆H > 0 Y = KNO3 e m2 = 60g → não será dissolvido IIIITTTTAAAA ---- ((((4444ºººº DDDDiiiiaaaa)))) DDDDeeeezzzzeeeemmmmbbbbrrrroooo////2222000000005555
  • 10. OOOOBBBBJJJJEEEETTTTIIIIVVVVOOOO Experimento 2. Correta: A solubilidade de NaClO3 é 120g/100g de H2O a 40°C Sendo: X = NaClO3 e m1 = 40g → solução insaturada ∆H > 0 Y = NaClO3 e m2 = 40g → solução insaturada ∆H > 0 Portanto, todo o sal acrescentado dissolve-se. Experimento 3. Incorreta: A solubilidade de NaCl é aproximadamente 30g/100g de H2O a 40°C Sendo: X = NaCl e m1 = 10g → solução insaturada ∆H ≅ 0 Y = NaCl e m2 = 10g → solução insaturada ∆H ≅ 0 Toda a massa de sal dissolve-se e o ∆H é aproxi- madamente zero. Experimento 4. Incorreta: Sendo: X = KNO3 e m1 = 60g → solução saturada ∆H > 0 Y = NaClO3 e m2 = 60g → solução insaturada ∆H > 0 Toda a massa de sal adicionado dissolve-se e o ∆H de dissolução é maior que zero. Experimento 5. Incorreta: A solubilidade de NH4Cl é aproximadamente 40g/100g de H2O a 40°C Sendo: X = KNO3 e m1 = 60g → solução saturada ∆H > 0 Y = NH4Cl e m2 = 60g → solução saturada, com excesso ∆H > 0 IIIITTTTAAAA ---- ((((4444ºººº DDDDiiiiaaaa)))) DDDDeeeezzzzeeeemmmmbbbbrrrroooo////2222000000005555
  • 11. OOOOBBBBJJJJEEEETTTTIIIIVVVVOOOO 9 BBBB A figura mostra cinco curvas de distribuição de velo- cidade molecular para diferentes gases (I, II, III, IV e V) a uma dada temperatura. Assinale a opção que relaciona CORRETAMENTE a curva de distribuição de velocidade molecular a cada um dos gases. a) I = H2 , II = He , III = O2 , IV = N2 e V = H2O. b) I = O2 , II = N2, III = H2O , IV = He e V = H2. c) I = He , II = H2 , III = N2 , IV= O2 e V = H2O. d) I = N2 , II = O2 , III = H2 , IV = H2O e V = He. e) I = H2O, II = N2 , III = O2 , IV = H2 e V = He. Resolução Quanto maior a massa molecular do gás, menor a velocidade média molecular. MMO2 > MMN2 > MMH 2 O > MMHe > MMH2 Foram dadas as curvas de Gauss de distribuição de velocidade. Podemos concluir que vO2 < vN2 < vH 2 O < vHe < vH2 I → O2 II → N2 III → H2O IV → He V → H2 IIIITTTTAAAA ---- ((((4444ºººº DDDDiiiiaaaa)))) DDDDeeeezzzzeeeemmmmbbbbrrrroooo////2222000000005555
  • 12. OOOOBBBBJJJJEEEETTTTIIIIVVVVOOOO 10 EEEE Considere as respectivas reações químicas represen- tadas pelas seguintes equações químicas: I. 2KMnO4 +3H2SO4 + 5H2O2 → 1X + 2Y + 8H2O + 5O2 II. 4 CrO3 + 6 H2SO4 → 2Z +6 H2O + 3O2 III.2K2Cr2O7+10H2SO4→4KHSO4+2W + 8H2O + 3O2 Em relação às equações químicas I, II e III é CORRETO afirmar que a) o produto X é KHSO4. b) o produto Y é Mn(SO4)2. c) o produto Z é CrSO4. d) o peróxido de hidrogênio atua como agente oxi- dante. e) os produtos Z e W representam o mesmo composto químico. Resolução I) 2KMnO4 + 3H2SO4 + 1– 0 + 5H2O2 → K2SO4 + 2MnSO4 + 8H2O + 5O2 X Y oxidação H2O2: agente redutor II) 4CrO3 + 6H2SO4 → 2Cr2(SO4)3 + 6H2O + 3O2 Z III) 2K2Cr2O7 + 10H2SO4 → → 4KHSO4 + 2Cr2(SO4)3 + 8H2O + 3O2 W Os produtos Z e W representam o mesmo composto. IIIITTTTAAAA ---- ((((4444ºººº DDDDiiiiaaaa)))) DDDDeeeezzzzeeeemmmmbbbbrrrroooo////2222000000005555
  • 13. OOOOBBBBJJJJEEEETTTTIIIIVVVVOOOO 11 DDDD Assinale a opção que apresenta a substância que pode exibir comportamento de cristal líquido, nas condições ambientes. Resolução No estado cristalino líquido, existe uma ordem mole- cular menor do que num sólido, no entanto, maior do que num líquido comum. Compostos que podem formar cristais líquidos têm moléculas compridas e razoavelmente rígidas. A substância que pode exibir comportamento de cristal líquido, nas condições ambientes, é: A substância da alternativa e, embora seja uma molé- cula de cadeia longa, tem menos interações dipolo- dipolo. CH3O — — CH = N — — CH2 — CH2 — CH2 — CH3 COOH COOCH3 CH3O — — CH = N — — CH2 — CH2 — CH2 — CH3d) CH3 — CH2 — CH2 — CH2 — CH2 — CH2 — CH2 — CH2 — CH2 — CH2 — OHe) c) — CH2COONaa) b) CH3 — CH2 — C — CH2 — C — CH2 — CH3 CH3 CH3 CH3 CH3 IIIITTTTAAAA ---- ((((4444ºººº DDDDiiiiaaaa)))) DDDDeeeezzzzeeeemmmmbbbbrrrroooo////2222000000005555
  • 14. OOOOBBBBJJJJEEEETTTTIIIIVVVVOOOO 12 CCCC Considere quatro séries de experimentos em que quatro espécies químicas (X, Y, Z e W) reagem entre si, à pressão e temperatura constantes. Em cada série, fixam-se as concentrações de três espécies e varia-se a concentração (C0) da quarta. Para cada série, de- termina-se a velocidade inicial da reação (v0) em cada experimento. Os resultados de cada série são apre- sentados na figura, indicados pelas curvas X, Y, Z e W, respectivamente. Com base nas informações for- necidas, assinale a opção que apresenta o valor CORRETO da ordem global da reação química. a) 3 b) 4 c) 5 d) 6 e) 7 Resolução Considerando a reação X + Y + Z + W → produtos, a equação da velocidade pode ser expressa por: v = k [X]a . [Y]b . [Z]c . [W]d Aplicando logaritmo em ambos os membros, temos: log v = log k + a log [X] + b log [Y] + c log [Z] + d log [W] Fixando-se as concentrações de três espécies e variando a concentração da quarta espécie poderíamos escrever 4 equações: I) log v = a log [X] + k1 II) log v = b log [Y] + k2 III) log v = c log [Z] + k3 IV) log v = d log [W] + k4 k1 = k2 = k3 = – 1,0; k4 = – 0,8 Nas equações, a, b, c, d, são os coeficientes angulares das 4 curvas fornecidas no gráfico log v0 x log C0. Para X, temos a = 0 Para Y, temos b = tg α1 = = = 1 Para Z, temos c = tg α2 = = = 2 Como as curvas W e Z são paralelas, apresentam o mesmo coeficiente angular ∴ d = 2 A equação da velocidade pode ser expressa por v = k [X]0 . [Y]1 . [Z]2 . [W]2 A ordem global da reação será: 0 + 1 + 2 + 2 = 5 0,4 –––– 0,2 (– 0,6) – (– 1,0) ––––––––––––– (– 0,8) – (1,0) 0,2 –––– 0,2 (– 0,8) – (– 1,0) ––––––––––––– (– 0,8) – (1,0) IIIITTTTAAAA ---- ((((4444ºººº DDDDiiiiaaaa)))) DDDDeeeezzzzeeeemmmmbbbbrrrroooo////2222000000005555
  • 15. OOOOBBBBJJJJEEEETTTTIIIIVVVVOOOO 13 EEEE Considere soluções de SiCl4/CCl4 de frações molares variáveis, todas a 25° C. Sabendo que a pressão de va- por do CCl4 a 25° C é igual a 114,9 mmHg, assinale a opção que mostra o gráfico que melhor representa a pressão de vapor de CCl4 (PCCl4 ) em função da fração molar de SiCl4 no líquido (X1 SiCl4 ) . IIIITTTTAAAA ---- ((((4444ºººº DDDDiiiiaaaa)))) DDDDeeeezzzzeeeemmmmbbbbrrrroooo////2222000000005555
  • 16. OOOOBBBBJJJJEEEETTTTIIIIVVVVOOOO Resolução Admitindo CCl4 puro (X’SiCl4 = 0), a pressão de vapor do CCl4 a 25°C vale 114,9mmHg. Admitindo SiCl4 puro (X’SiCl4 = 1), a pressão de vapor do CCl4 será 0 (zero). As alternativas a, b, c são eliminadas. Admitindo uma solução ideal (os dois líquidos com- pletamente miscíveis entre si), a pressão de vapor do CCl4 é dada pela Lei de Raoult: PCCl4 = P0 CCl4 . XCCl4 em que P0 CCl4 é a pressão de vapor do CCl4 puro na mesma temperatura e XCCl4 , a fração do CCl4 na mis- tura. Como a fração do CCl4 vai diminuindo à medida que a fração do SiCl4 aumenta, o gráfico PCCl4 / X’SiCl4 será uma reta decrescente. IIIITTTTAAAA ---- ((((4444ºººº DDDDiiiiaaaa)))) DDDDeeeezzzzeeeemmmmbbbbrrrroooo////2222000000005555
  • 17. OOOOBBBBJJJJEEEETTTTIIIIVVVVOOOO 14 CCCC Um recipiente fechado, mantido a volume e tempe- ratura constantes, contém a espécie química X no estado gasoso a pressão inicial Po. Esta espécie decompõe-se em Y e Z de acordo com a seguinte equa- ção química: X(g) → 2Y(g) + 1/2 Z(g). Admita que X, Y e Z tenham comportamento de gases ideais. Assinale a opção que apresenta a expressão CORRETA da pressão (P) no interior do recipiente em função do andamento da reação, em termos da fração α de moléculas de X que reagiram. a) P = [1 + (1/2)α] Po b) P = [1 + (2/2)α] Po c) P = [1 + (3/2)α] Po d) P = [1+(4/2)α] ]Po e) P = [1 + (5/2)α] Po Resolução A reação ocorre a volume e temperatura constantes, e os participantes da reação são todos gasosos, logo a pressão total do sistema depende da quantidade em mol. Considere a tabela para a variação da pressão do sistema. Seja P a pressão total do sistema em um instante t. P = P0 (1 – α) + 2αP0 + αP0 P = P0 1 + α΅3––– 2΄ 1 ––– 2 1 ––– αP0 2 2αP0P0(1 – α)instante t 1 ––– αP0 2 2αP0αP0reage/forma 00P0início 1X(g) → 2Y(g) + ––– Z(g) 2 IIIITTTTAAAA ---- ((((4444ºººº DDDDiiiiaaaa)))) DDDDeeeezzzzeeeemmmmbbbbrrrroooo////2222000000005555
  • 18. OOOOBBBBJJJJEEEETTTTIIIIVVVVOOOO 15 AAAA Um elemento galvânico é constituído pelos eletrodos abaixo especificados, ligados por uma ponte salina e conectados a um multímetro de alta impedância. Eletrodo a: Placa de chumbo metálico mergulhada em uma solução aquosa 1 mol L–1de nitrato de chumbo. Eletrodo b: Placa de níquel metálico mergulhada em uma solução aquosa 1 mol L–1 de sulfato de níquel. Após estabelecido o equilíbrio químico nas condições- padrão, determina-se a polaridade dos eletrodos. A seguir, são adicionadas pequenas porções de KI sólido ao Eletrodo a, até que ocorra a inversão de polaridade do elemento galvânico. Dados eventualmente necessários: Produto de solubilidade de PbI2: Kps (PbI2) = 8,5 x 10–9 Potenciais de eletrodo em relação ao eletrodo padrão de hidrogênio nas condições-padrão: E0 Pb/Pb2+ = – 0,13V ; E0 Ni/Ni2+ = – 0,25V; E0 I–/I2 = 0,53V Assinale a opção que indica a concentração CORRETA de KI, em mol L–1, a partir da qual se observa a inversão de polaridade dos eletrodos nas condições-padrão. a) 1 x 10–2 b) 1 x 10–3 c) 1 x 10–4 d) 1 x 10–5 e) 1 x 10–6 Resolução (I) Cálculo do ∆E0 inicial da pilha: Eletrodo a ᮍ Cátodo: Pb2+(aq) + 2e– →← Pb0(s) – 0,13V Eletrodo b ᮎ Ânodo: Ni0(s) →← 2e–+ Ni2+(aq) + 0,25V ––––––––––––––––––––––––––––––––– Equação global Pb2+(aq) + Ni0(s) →← Pb0(s) + Ni2+(aq) (II)Observa-se a inversão de polaridade dos eletrodos nas condições-padrão quando E Pb2+ < – 0,25V Pela Equação de Nernst, temos: E = E0 – . log – 0,25 = – 0,13 – 0,02955 . log . 4,06 = log ∴ [Pb2+] ≅ 10–4 Cálculo da concentração de I– : KPS = [Pb2+] [ I –] 2 8,5 . 10–9 = 10–4 [ I –] 2 Logo: [KI] ≅ 10–2mol/L [ I –] ≅ 10–2mol/L mol ––––– L 1 ––––––– [Pb2+] 1 ––––––– [Pb2+] 1 ––––––– [Pb2+] 0,0591 ––––––– 2 ∆E0 = +0,12V IIIITTTTAAAA ---- ((((4444ºººº DDDDiiiiaaaa)))) DDDDeeeezzzzeeeemmmmbbbbrrrroooo////2222000000005555
  • 19. OOOOBBBBJJJJEEEETTTTIIIIVVVVOOOO 16 EEEE São dadas as semi-equações químicas seguintes e seus respectivos potenciais elétricos na escala do eletrodo de hidrogênio nas condições-padrão: I. Cl2(g)+2e– → ← 2Cl–(aq); E0 I = +1,358V II. Fe2+(aq) + 2e– → ← Fe(s) ; E0 II = –0,447V III.Fe3+(aq) + 3e– → ← Fe(s) ; E0 III = –0,037V IV.Fe3+(aq) + e– → ← Fe2+ (aq) ; E0 IV = +0,771V V. O2(g)+4H+(aq)+4e– → ← 2H2O(l) ; E0 V = + 1,229V Com base nestas informações, assinale a opção que contém a afirmação CORRETA, considerando as con- dições-padrão. a) A formação de FeCl2 a partir de Fe fundido e Cl2 gasoso apresenta ∆H > 0. b) Tanto a eletrólise ígnea do FeCl2(s) quanto a do FeCl3(s), quando realizadas nas mesmas condições experimentais, produzem as mesmas quantidades em massa de Fe(s) . c) Uma solução aquosa de FeCl2 reage com uma solução aquosa de ácido clorídrico, gerando H2(g). d) Borbulhando Cl2(g) em uma solução aquosa de Fe2+, produz-se 1 mol de Fe3+ para cada mol de Cl– em solução. e) Fe2+ tende a se oxidar em solução aquosa ácida quando o meio estiver aerado. Resolução Fe2+ tende a oxidar-se em solução aquosa ácida quan- do o meio estiver aerado. 4Fe2+(aq) →← 4Fe3+(aq) + 4e– E 0 oxid = – 0,771V O2(g) + 4H+(aq) + 4e– →← 2H2O(l) E 0 red = + 1,229V –––––––––––––––––––––––––––––––––––––––––––––––––––––––––––– 4Fe2+(aq)+O2(g)+4H+(aq) →← 4Fe3+(aq)+2H2O(l) ∆E = 0,458V IIIITTTTAAAA ---- ((((4444ºººº DDDDiiiiaaaa)))) DDDDeeeezzzzeeeemmmmbbbbrrrroooo////2222000000005555
  • 20. OOOOBBBBJJJJEEEETTTTIIIIVVVVOOOO 17 CCCC Duas células (I e II) são montadas como mostrado na figura. A célula I contém uma solução aquosa 1 mol L–1 em sulfato de prata e duas placas de platina. A célula II contém uma solução aquosa 1 mol L–1 em sulfato de cobre e duas placas de cobre. Uma bateria fornece uma diferença de potencial elétrico de 12 V entre os eletrodos Ia e IIb, por um certo intervalo de tempo. Assinale a opção que contém a afirmativa ERRADA em relação ao sistema descrito. a) Há formação de O2(g) no eletrodo Ib. b) Há um aumento da massa do eletrodo Ia. c) A concentração de íons Ag+ permanece constante na célula I. d) Há um aumento de massa do eletrodo lIa. e) A concentraçao de íons Cu2+ permanece constante na célula lI. Resolução Célula I: cátodo (Ia): Ag+(aq) + e– → Ag(s) ânodo (Ib): H2O(l) → 2e– + 1/2 O2(g) + 2H+(aq) No eletrodo Ib, haverá a formação de O2(g) devido à oxidação da água. No eletrodo Ia, haverá a formação de Ag(s) devido à redução dos íons Ag+(aq), diminuindo a concentração destes na célula I. Célula II: cátodo (IIa): Cu2+(aq) + 2e– → Cu(s) ânodo (IIb): Cu(s) → 2e– + Cu2+(aq) No eletrodo IIa, haverá a formação de Cu(s) devido à redução dos íons Cu2+(aq). IIIITTTTAAAA ---- ((((4444ºººº DDDDiiiiaaaa)))) DDDDeeeezzzzeeeemmmmbbbbrrrroooo////2222000000005555
  • 21. OOOOBBBBJJJJEEEETTTTIIIIVVVVOOOO No eletrodo IIb, ocorrerá a oxidação de Cu(s) a íons Cu2+(aq), mantendo constante a concentração destes na célula II. IIIITTTTAAAA ---- ((((4444ºººº DDDDiiiiaaaa)))) DDDDeeeezzzzeeeemmmmbbbbrrrroooo////2222000000005555
  • 22. OOOOBBBBJJJJEEEETTTTIIIIVVVVOOOO 18 DDDD Considere as afirmações abaixo, todas relacionadas a átomos e íons no estado gasoso: I. A energia do íon Be2+, no seu estado fundamental, é igual à energia do átomo de He neutro no seu estado fundamental. II. Conhecendo a segunda energia de ionização do átomo de He neutro, é possível conhecer o valor da afinidade eletrônica do íon He2+. III.Conhecendo o valor da afinidade eletrônica e da primeira energia de ionização do átomo de Li neutro, é possível conhecer a energia envolvida na emissão do primeiro estado excitado do átomo de Li neutro para o seu estado fundamental. IV.A primeira energia de ionização de íon H – é menor do que a primeira energia de ionização do átomo de H neutro. V. O primeiro estado excitado do átomo de He neutro tem a mesma configuração eletrônica do primeiro estado excitado do íon Be2+. Então, das afirmações acima, estão CORRETAS a) apenas I e III. b) apenas I, II e V. c) apenas I e IV. d) apenas II, IV e V. e) apenas III e V. Resolução I) Incorreta A energia do íon Be 2+, que possui 4 protóns e 2 elétrons (maior atração nuclear) é diferente da do átomo de He neutro, que possui 2 prótons e 2 elé- trons. II) Correta He+ + energia → He2+ + e– (I) He2+ + e– → He+ + energia (II) Conhecendo a segunda energia de ionização do átomo de He (I), é possível conhecer o valor da afinidade eletrônica do íon He2+ (II). III)Incorreta Li + e– → Li – + energia (I) Li + energia → Li+ + e– (II) Conhecendo o valor da afinidade eletrônica (I) e da primeira energia de ionização (II) do átomo de Li neutro, não é possível conhecer a energia envolvida na emissão do primeiro estado excitado do átomo de Li neutro para o seu estado fundamental, no qual o elétron volta de um subnível mais energético para um subnível menos energético. Li (excitado) → Li (fundamental) IV)Correta H– + energia → H + e– (I) H + energia → H+ + e– (II) A primeira energia de ionização de íon H– (I) é menor do que a primeira energia de ionização do átomo de IIIITTTTAAAA ---- ((((4444ºººº DDDDiiiiaaaa)))) DDDDeeeezzzzeeeemmmmbbbbrrrroooo////2222000000005555
  • 23. OOOOBBBBJJJJEEEETTTTIIIIVVVVOOOO H neutro (II), pois esta é a segunda energia de ionização do íon H–. V) Correta O primeiro estado excitado do átomo de He neutro tem a mesma configuração eletrônica do primeiro estado excitado do íon Be2+, pois as duas espécies possuem 2 elétrons, e sua configuração não-excitada é 1s2. 19 BBBB Uma reação química hipotética é representada pela seguinte equação: X(g) + Y(g) → 3Z(g). Considere que esta reação seja realizada em um cilindro provido de um pistão, de massa desprezível, que se desloca sem atrito, mantendo-se constantes a pressão em 1 atm e a temperatura em 25°C. Em relação a este sistema, são feitas as seguintes afirmações: I. O calor trocado na reação é igual à variação de entalpia. II. O trabalho realizado pelo sistema é igual a zero. III.A variação da energia interna é menor do que a variação da entalpia. IV.A variação da energia interna é igual a zero. V. A variação da energia livre de Gibbs é igual à variação de entalpia. Então, das afIrmações acima, estão CORRETAS a) apenas I, II e IV. b) apenas I e III. c) apenas II e V. d) apenas III e IV. e) apenas III, IV e V. Resolução I. Correta. X(g) + Y(g) → 3Z(g) 2 mol 3 mol ∆H = QP , pressão constante II. Errada. τ ≠ 0, τ = ∆n R T, τ = 1 . R T III.Correta. ∆H = ∆U + τ ∴ ∆H = ∆U + R T IV.Errada. ∆U ≠ 0, ∆U = ∆H – R T Observação – Quando não ocorre reação quími- ca, se a temperatura permanecer constante, a varia- ção de energia interna é nula. Não é o nosso caso. V. Errada. ∆G = ∆H – T ∆S IIIITTTTAAAA ---- ((((4444ºººº DDDDiiiiaaaa)))) DDDDeeeezzzzeeeemmmmbbbbrrrroooo////2222000000005555
  • 24. OOOOBBBBJJJJEEEETTTTIIIIVVVVOOOO 20 EEEE A figura apresenta cinco curvas (I, lI, III, IV e V) da concentração de uma espécie X em função do tempo. Considerando uma reação química hipotética re- presentada pela equação X(g) → Y(g), assinale a opção CORRETA que indica a curva correspondente a uma reação química que obedece a uma lei de velocidade de segunda ordem em relação à espécie X. a) Curva I b) Curva II c) Curva III d) Curva IV e) Curva V Resolução Para uma reação de segunda ordem, a equação de velocidade é dada pela expressão: X(g) → Y(g) v= k [X] 2 Escrevendo a velocidade em função da concentração e do tempo, temos: v = = k [X]2 = k d t Integrando, temos: = kt ∴ [X] . t = (equação de uma hipérbole) Dentre as curvas fornecidas, a hipérbole corresponde à curva V. Confirmando com os valores fornecidos no gráfico: 1 ––– k 1 ––– [X] d [X] – ––––– [X]2 – d [X] –––––– dt IIIITTTTAAAA ---- ((((4444ºººº DDDDiiiiaaaa)))) DDDDeeeezzzzeeeemmmmbbbbrrrroooo////2222000000005555
  • 25. OOOOBBBBJJJJEEEETTTTIIIIVVVVOOOO Ponto A [X] . t = 5 mmol L– 1 . 2 s = 10 . mmol . L–1 . s Ponto B [X] . t = 4 mmol L– 1 . 2,5 s = 10 mmol . L– 1 . s Ponto C [X] . t = 2 mmol L– 1 . 5 s = 10 mmol . L– 1 . s As questões dissertativas, numeradas de 21 a 30, devem ser resolvidas e respondidas no caderno de soluções. 21Considere as informações abaixo: a) PbCrO4(s) é um sólido amarelo que é solúvel em água quente. b) AgCl(s) forma um cátion de prata solúvel em solução aquosa de NH3. c) O sólido branco PbCl2(s) é solúvel em água quente, mas os sólidos brancos AgCl(s) e Hg2Cl2(s) não o são. d) Uma solução aquosa contendo o cátion de prata do item b), quando acidulada com HCl, forma o sólido AgCl(s). e) Hg2Cl2(s) forma uma mistura insolúvel contendo Hg(l), que tem cor prata, e HgNH2Cl(s), que é preto, em solução aquosa de NH3. Uma amostra sólida consiste em uma mistura de cloretos de Ag+, Hg2 2+ e Pb2+. Apresente uma seqüên- cia de testes experimentais para identificar os íons Ag+, Hg2 2+ e Pb2+ presentes nesta amostra. Resolução Considerando a amostra sólida formada pelos sais AgCl, Hg2Cl2 e PbCl2, podemos solubilizar o PbCl2 adicionando água quente. Filtrando o sistema, teremos uma solução contendo cloreto de Pb2+ solúvel (em água quente) que poderá ser identificado adicionando uma solução contendo íons CrO4 2– . Pb2+(aq) + CrO4 2– (aq) → PbCrO4(s) ↓amarelo Ao resíduo sólido retido no filtro (Hg2Cl2 + AgCl), acres- centamos solução aquosa de amônia que irá solubilizar o AgCl(s)(reação de complexação): AgCl(s) + 2NH3(aq) → Ag(NH3)2 + (aq) + Cl–(aq) Filtrando esse novo sistema e adicionando HCl ao filtrado (excesso de Cl–), haverá precipitação de AgCl, de cor branca. Ag(NH3)2 + (aq) + Cl – (aq) → AgCl(s) ↓+ 2NH3(aq) ppt No resíduo da última filtração, observamos mercúrio líquido de cor prata e HgNH2Cl(s) preto. IIIITTTTAAAA ---- ((((4444ºººº DDDDiiiiaaaa)))) DDDDeeeezzzzeeeemmmmbbbbrrrroooo////2222000000005555
  • 26. OOOOBBBBJJJJEEEETTTTIIIIVVVVOOOO 22Calcule o valor do potencial elétrico na escala do ele- trodo de hidrogênio nas condições-padrão (E0) da semi- equação química CuI(s) + e–(CM) ← → Cu(s) + I –(aq). Dados eventualmente necessários: Produto de solu- bilidade do CuI(s): Kps (CuI) = 1,0 x 10–12 Semi-equações químicas e seus respectivos potenciais elétricos na escala do eletrodo de hidrogênio nas condições-padrão (E0): I. Cu2+(aq) + e– (CM) ← → Cu+(aq) ; E 0 I = 0,15 V II. Cu2+(aq) + 2e–(CM) ← → Cu(s) ; E 0 II =0,34V III. Cu+(aq) + e– (CM) ← → Cu(s) ; E 0 III = 0,52 V IV. I2(s) + 2e– (CM) ← → 2I–(aq) ; E 0 IV = 0,54 V Resolução A semi-equação química fornecida: CuI(s) + e–(CM) →← Cu(s) + I– (aq) E = ? redução representa a redução do íon Cu+ no composto CuI. O dado número III fornece a semi-equação: Cu+(aq) + e–(CM) →← Cu(s) E0 III = 0,52V Como a equação fornecida apresenta o íon Cu+ precipitado na forma de CuI, calcula-se a concentração final de Cu+ por meio do PS. CuI(s) →← Cu+(aq) + I– (aq) x x PS = [Cu+] . [I– ] }x = 1,0 . 10–6 mol/L 1,0 . 10–12 = x2 Cálculo do E: Pela Equação de Nernst, temos: E = E0 III – . log E = 0,52 – 0,0591 . log E = + 0,52 – 0,0591 . (6) E = + 0,52 – 0,3546 = 0,1654 E = 0,1654V 1 –––––– 10–6 1 –––––– [Cu+] 0,0591 –––––––– 1 01+ IIIITTTTAAAA ---- ((((4444ºººº DDDDiiiiaaaa)))) DDDDeeeezzzzeeeemmmmbbbbrrrroooo////2222000000005555
  • 27. OOOOBBBBJJJJEEEETTTTIIIIVVVVOOOO 23Esboce graficamente o diagrama de fases (pressão versus temperatura) da água pura (linhas cheias). Neste mesmo gráfico, esboce o diagrama de fases de uma solução aquosa 1 moI kg–1 em etilenoglicol (linhas tracejadas). Resolução IIIITTTTAAAA ---- ((((4444ºººº DDDDiiiiaaaa)))) DDDDeeeezzzzeeeemmmmbbbbrrrroooo////2222000000005555
  • 28. OOOOBBBBJJJJEEEETTTTIIIIVVVVOOOO 24Uma reação química genérica pode ser representada pela seguinte equação: A(s) ← → B(s) + C(g). Sabe-se que, na temperatura Teq, esta reação atinge o equilíbrio químico, no qual a pressão parcial de C é dada por PC,eq . Quatro recipientes fechados (I, II, III e IV), mantidos na temperatura Teq, contêm as misturas de substâncias e as condições experimentais especificadas abaixo: I. A(s) + C(g) ; PC,I < PC,eq ll. A(s) + B(s) ; PC,II = 0 III.A(s) + C(g) ; PC,III >>> Pc,eq IV.B(s) + C(g) ; PC,IV > PC,eq Para cada um dos recipientes, o equilíbrio químico ci- tado pode ser atingido? Justifique suas respostas. Resolução Como a pressão de C no re- cipiente I é menor que a pres- são de C no equilíbrio, a substância A irá transformar-se em B e C até que a pressão de C seja PC,eq (o equilíbrio pode ser atingido). Irá ocorrer a reação, A(s) →← B (s) + C(g) A substância A (sólido) irá produzir mais B (sólido) e C (gás) até que a pressão de C seja PC,eq (o equilíbrio pode ser atingido). Para que o equilíbrio fosse atingido, a pressão de C (PC,III) deveria diminuir, o que é im- possível por não haver a substância B (sólida) no reci- piente. Como a pressão de C no re- cipiente é maior que a sua pressão no equilíbrio, C deverá reagir com B sólido, produzindo a substância A (sólida) e o equilíbrio pode ser atingido. Obs.: nas experiências I, II e IV, admite-se haver rea- gente em quantidade suficiente para estabelecer o equilíbrio. IIIITTTTAAAA ---- ((((4444ºººº DDDDiiiiaaaa)))) DDDDeeeezzzzeeeemmmmbbbbrrrroooo////2222000000005555
  • 29. OOOOBBBBJJJJEEEETTTTIIIIVVVVOOOO 25Uma substância A apresenta as seguintes proprie- dades: Temperatura de fusão a 1 atm = – 20 °C Temperatura de ebulição a 1 atm = 85 °C Variação de entalpia de fusão = 180 J g–1 Variação de entalpia de vaporização = 500 J g– 1 Calor específico de A(s) = 1,0J g–1 °C–1 Calor específico de A(l)= 2,5J g–1 °C–1 Calor específico de A(g) = 0,5J g–1 °C–1 À pressão de 1 atm, uma amostra de 25g de substância A é aquecida de – 40°C até 100°C, a uma velocidade constante de 450 J min–1. Considere que todo calor fornecido é absorvido pela amostra. Construa o gráfico de temperatura (°C) versus tempo (min) para todo o processo de aquecimento considerado, indicando claramente as coordenadas dos pontos iniciais e finais de cada etapa do processo. Mostre os cálculos necessários. Resolução I) Aquecimento de – 40°C a – 20°C: Q1 = m c ∆θ Q1 = 25 . 1 . (20) Q1 = 500J Assim: ∆t1 = = = 1,1 min Em que Pot é potência constante com que o calor é fornecido à amostra. II) Fusão da substância A: Q2 = m LF Em que LF é a variação de entalpia de fusão. Q2 = 25 . 180 Q2 = 4500J ∴ ∆t2 = = = 10min III) Aquecimento da substância no estado líquido: Q3 = m c ∆θ Q3 = 25 . 2,5 . (85 – (– 20)) Q3 = 6562,5J ∴ ∆t3 = = = 14,6min IV) Vaporização da substância A: Q4 = m LV Em que LV é a variação da entalpia de vaporização. Q4 = 25 . 500 6562,5 –––––––– 450 Q3 –––– Pot 4500 ––––– 450 Q2 –––– Pot 500 –––– 450 Q1 –––– Pot IIIITTTTAAAA ---- ((((4444ºººº DDDDiiiiaaaa)))) DDDDeeeezzzzeeeemmmmbbbbrrrroooo////2222000000005555
  • 30. OOOOBBBBJJJJEEEETTTTIIIIVVVVOOOO Q4 = 12500J ∴ ∆t4 = = = 27,8min V) Aquecimento do vapor: Q5 = m c ∆θ Q5 = 25 . 0,5 (15) Q5 = 187,5J ∴ ∆t5 = = ≅ 0,42min Dessa maneira, o gráfico pedido é: Nota: O gráfico não está em escala. 187,5 –––––––– 450 Q5 –––– Pot 12500 –––––––– 450 Q4 –––– Pot IIIITTTTAAAA ---- ((((4444ºººº DDDDiiiiaaaa)))) DDDDeeeezzzzeeeemmmmbbbbrrrroooo////2222000000005555
  • 31. OOOOBBBBJJJJEEEETTTTIIIIVVVVOOOO 26Para cada um dos processos listados abaixo, indique se a variação de entropia será maior, menor ou igual a zero. Justifique suas respostas. a) N2(g,1 atm, T = 300 K) → N2(g,0,1 atm, T = 300 K) b) C (grafite) → C(diamante) c) solução supersaturada → solução saturada d) sólido amorfo → sólido cristalino e) N2(g) → N2 (g, adsorvido em sílica) Resolução A entropia é uma grandeza termodinâmica que mede o grau de desordem de um sistema. maior desordem → maior entropia a) N2(g, 1 atm, T = 300K) → N2 (g, 0,1 atm, T = 300K) S1 S2 S2 > S1 : ∆S > 0, pois houve expansão de volume (diminuição de pressão), aumentando a desordem. b) C(grafita) → C(diamante) S1 S2 menos mais organizado organizado S1 > S2 : ∆S < 0 c) Solução supersaturada → solução saturada S1 S2 S1 > S2 : ∆S < 0, parte das partículas que estavam na solução supersaturada cristalizaram, portanto, dimi- nuindo a entropia total do sistema. d) sólido amorfo → sólido cristalino partículas em partículas em desordem ordem S1 S2 S1 > S2 : ∆S < 0 e) N2(g) → N2 (g, adsorvido em sílica) S1 S2 S1 > S2 : ∆S < 0 IIIITTTTAAAA ---- ((((4444ºººº DDDDiiiiaaaa)))) DDDDeeeezzzzeeeemmmmbbbbrrrroooo////2222000000005555
  • 32. OOOOBBBBJJJJEEEETTTTIIIIVVVVOOOO 27A equação química hipotética A → D ocorre por um me- canismo que envolve as três reações unimoleculares abaixo (I, II e III). Nestas reações, ∆Hi representa as variações de entalpia, e Eai, as energias de ativação. I. A → B; rápida, ∆HI, EaI II. B → C; lenta, ∆HII, EaII III.C → D; rápida, ∆HIII, EaIII Trace a curva referente à energia potencial em função do caminho da reação A → D, admitindo que a reação global A → D seja exotérmica e considerando que: ∆HII >∆HI > 0; EaI <EaIll. Resolução IIIITTTTAAAA ---- ((((4444ºººº DDDDiiiiaaaa)))) DDDDeeeezzzzeeeemmmmbbbbrrrroooo////2222000000005555
  • 33. OOOOBBBBJJJJEEEETTTTIIIIVVVVOOOO 28São realizadas reações químicas do acetileno com áci- do clorídrico, ácido cianídrico, ácido acético e cloro, nas proporções estequiométricas de 1:1. a) Mostre as equações químicas que representam cada uma das reações químicas especificadas. b) Indique quais dos produtos formados podem ser uti- lizados como monômeros na síntese de polímeros. c) Dê os nomes dos polímeros que podem ser formados a partir dos monômeros indicados no item b). Resolução H Cl | | a) H — C ≡ C — H + HCl → H — C = C — H H CN | | H — C ≡ C — H + HCN → H — C = C — H O H — C ≡ C — H + C — CH3 → HO O C — CH3 H O | | → H — C = C — H Cl Cl | | H — C ≡ C — H + Cl2 → H — C = C — H b) Todos os produtos são usados como monômeros. c) poli(cloreto de vinila) ou PVC poli(cianeto de vinila) ou poliacrilonitrilo ou orlon poli(acetato de vinila) ou PVA poli(1,2-dicloroetileno) IIIITTTTAAAA ---- ((((4444ºººº DDDDiiiiaaaa)))) DDDDeeeezzzzeeeemmmmbbbbrrrroooo////2222000000005555
  • 34. OOOOBBBBJJJJEEEETTTTIIIIVVVVOOOO 29São descritos, a seguir, dois experimentos e respec- tivas observações envolvendo ossos limpos e secos provenientes de uma ave. I. Um osso foi imerso em uma solução aquosa 10% (v/v) em ácido fórmico. Após certo tempo, observou-se que o mesmo havia se tornado flexível. II. Um outro osso foi colocado em uma cápsula de porcelana e submetido a aquecimento em uma cha- ma de bico de Bunsen. Após um longo período de tempo, observou-se que o mesmo se tornou frágil e quebradiço. a) Explique as observações descritas nos dois expe- rimentos. b) Baseando-se nas observações acima, preveja o que acontecerá se um terceiro osso limpo e seco for imerso em uma solução aquosa 1 mg L–1 em fluore- to de sódio e, a seguir, em uma solução aquosa a 10% (v/v) em ácido fórmico. Justifique a sua res- posta. Resolução Experiência I: a) Com a retirada do cálcio, pela imersão em ácido, o osso se torna flexível devido à permanência de substância orgânica intersticial (responsável pela flexibilidade). O osso é constituído por um sal iônico, a hidro- xiapatita, de fórmula Ca10(PO4)6 (OH)2 e substân- cias orgânicas O Reação:Ca10(PO4)6(OH)2(s) + 2 HC (aq) → OH → Ca10(PO4)6(HCOO)2(aq) + 2H2O(l) Experiência II: Há eliminação, pelo aquecimento, do material orgânico intersticial – osseína – , restando apenas o sal inor- gânico, Ca10(PO4)6(OH)2, bem como a retirada da H2O, por vaporização, reduzindo a flexibilidade do osso, tor- nando-o mais frágil e quebradiço. b) O terceiro osso, imerso em solução de fluoreto de sódio, é fortalecido, impedindo que posteriormente o sal de flúor formado seja dissolvido com o ácido. Ca10(PO4)6(OH)2(s) + 2NaF(aq) → → Ca10(PO4)6F2(s) + 2NaOH(aq) resistente ao ácido IIIITTTTAAAA ---- ((((4444ºººº DDDDiiiiaaaa)))) DDDDeeeezzzzeeeemmmmbbbbrrrroooo////2222000000005555
  • 35. OOOOBBBBJJJJEEEETTTTIIIIVVVVOOOO 30Considere as seguintes espécies no estado gasoso: BF3, SnF – 3, BrF3, KrF4 e BrF5. Para cada uma delas, qual é a hibridização do átomo central e qual o nome da geometria molecular? Resolução BF3 24 elétrons = 12 pares de elétrons 3 pares de elétrons no átomo central: hibridização sp2 geometria molecular: plana trigonal SnF3 – 26 elétrons = 13 pares de elétrons 4 pares de elétrons: hibridização sp3 geometria molecular: piramidal BrF3 28 elétrons = 14 pares de elétrons 5 pares de elétrons: hibridização dsp3 geometria molecular: forma de T KrF4 36 elétrons = 18 pares de elétrons 6 pares de elétrons: hibridização d2sp3 geometria molecular: plana quadrada BrF5 42 elétrons = 21 pares de elétrons IIIITTTTAAAA ---- ((((4444ºººº DDDDiiiiaaaa)))) DDDDeeeezzzzeeeemmmmbbbbrrrroooo////2222000000005555
  • 36. OOOOBBBBJJJJEEEETTTTIIIIVVVVOOOO 6 pares de elétrons: hibridização d2sp3 geometria molecular: pirâmide de base quadrada CCCCOOOOMMMMEEEENNNNTTTTÁÁÁÁRRRRIIIIOOOO EEEE GGGGRRRRÁÁÁÁFFFFIIIICCCCOOOO Como já é tradicional, a prova de Química do ITA foi de nível elevadíssimo, com várias questões cujos assuntos não estão no programa do Ensino Médio. A distribuição dos assuntos foi totalmente irregular: 20 questões de físico-química, 6 de química inorgânica e 4 de química orgânica. IIIITTTTAAAA ---- ((((4444ºººº DDDDiiiiaaaa)))) DDDDeeeezzzzeeeemmmmbbbbrrrroooo////2222000000005555